Você está na página 1de 136

Teoria dos Números

Uma Introdução

Ismael Dourado & Cleber Pereira

1ª Ed - 2021
Sumário

Capítulo 1 Aritmética dos Inteiros 1


1.1 O conjunto dos Números Inteiros . . . . . . . . . . . . . 1
Exercícios . . . . . . . . . . . . . . . . . . . . . . . . . . . . . 4
1.2 Ordenação dos números inteiros . . . . . . . . . . . . . . 4
Exercícios . . . . . . . . . . . . . . . . . . . . . . . . . . . . . 9

Capítulo 2 Indução Matemática 11


2.1 Princípio de Indução . . . . . . . . . . . . . . . . . . . . 11
Exercícios . . . . . . . . . . . . . . . . . . . . . . . . . . . . . 17
2.2 Binômio de Newton . . . . . . . . . . . . . . . . . . . . 18
Exercícios . . . . . . . . . . . . . . . . . . . . . . . . . . . . . 23

Capítulo 3 Divisão Euclidiana 25


3.1 Divisibilidade . . . . . . . . . . . . . . . . . . . . . . . . 25
Exercícios . . . . . . . . . . . . . . . . . . . . . . . . . . . . . 31
3.2 O Máximo Divisor Comum . . . . . . . . . . . . . . . . 33
Exercícios . . . . . . . . . . . . . . . . . . . . . . . . . . . . . 39
3.3 Consequências do Máximo Divisor Comum . . . . . . . 40
Exercícios . . . . . . . . . . . . . . . . . . . . . . . . . . . . . 42
3.4 O Mínimo Múltiplo Comum . . . . . . . . . . . . . . . . 43
Exercícios . . . . . . . . . . . . . . . . . . . . . . . . . . . . . 45

Capítulo 4 Os Números Primos 47


4.1 Fatoração Primária . . . . . . . . . . . . . . . . . . . . . 47
Exercícios . . . . . . . . . . . . . . . . . . . . . . . . . . . . . 53
4.2 Determinação dos Fatores Primos . . . . . . . . . . . . . 54
Exercícios . . . . . . . . . . . . . . . . . . . . . . . . . . . . . 58
Capítulo 5 Funções 59
5.1 Função Multiplicativa . . . . . . . . . . . . . . . . . . . 59
Exercícios . . . . . . . . . . . . . . . . . . . . . . . . . . . . . 64
5.2 Função Divisor e Soma de Divisores . . . . . . . . . . . 64
Exercícios . . . . . . . . . . . . . . . . . . . . . . . . . . . . . 69
5.3 Função Maior Inteiro . . . . . . . . . . . . . . . . . . . . 69
Exercícios . . . . . . . . . . . . . . . . . . . . . . . . . . . . . 74

Capítulo 6 Equações Diofantinas 75


6.1 Soluções Inteiras . . . . . . . . . . . . . . . . . . . . . . 75
Exercícios . . . . . . . . . . . . . . . . . . . . . . . . . . . . . 78
6.2 Soluções Não Negativas . . . . . . . . . . . . . . . . . . 79
Exercícios . . . . . . . . . . . . . . . . . . . . . . . . . . . . . 85

Capítulo 7 Congruência dos Inteiros 86


7.1 Números Congruentes . . . . . . . . . . . . . . . . . . . 86
Exercícios . . . . . . . . . . . . . . . . . . . . . . . . . . . . . 91
7.2 Propriedades da Congruência . . . . . . . . . . . . . . . 92
Exercícios . . . . . . . . . . . . . . . . . . . . . . . . . . . . . 95
7.3 Congruência Linear . . . . . . . . . . . . . . . . . . . . . 96
Exercícios . . . . . . . . . . . . . . . . . . . . . . . . . . . . . 99
7.4 O Teorema Chinês dos Restos . . . . . . . . . . . . . . . 100
Exercícios . . . . . . . . . . . . . . . . . . . . . . . . . . . . . 106
7.5 Classes de Resíduos . . . . . . . . . . . . . . . . . . . . . 107
Exercícios . . . . . . . . . . . . . . . . . . . . . . . . . . . . . 109

Capítulo 8 Critérios de Divisibilidade 111


8.1 Representação dos Inteiros . . . . . . . . . . . . . . . . . 111
Exercícios . . . . . . . . . . . . . . . . . . . . . . . . . . . . . 114
8.2 Critérios de Divisibilidade . . . . . . . . . . . . . . . . . 115
Exercícios . . . . . . . . . . . . . . . . . . . . . . . . . . . . . 117

Capítulo 9 Fermat e Wilson 119


9.1 Teorema de Fermat . . . . . . . . . . . . . . . . . . . . . 119
Exercícios . . . . . . . . . . . . . . . . . . . . . . . . . . . . . 121
9.2 Teorema de Wilson . . . . . . . . . . . . . . . . . . . . . 122
Exercícios . . . . . . . . . . . . . . . . . . . . . . . . . . . . . 123

Capítulo 10 Euler 125


10.1 Função de Euler . . . . . . . . . . . . . . . . . . . . . . 125
Exercícios . . . . . . . . . . . . . . . . . . . . . . . . . . . . . 128
10.2 Teorema de Euler . . . . . . . . . . . . . . . . . . . . . . 129
Exercícios . . . . . . . . . . . . . . . . . . . . . . . . . . . . . 132
1
Aritmética dos Inteiros

Neste capítulo desenvolveremos a base que sustentará os capítulos


seguintes.

1.1 O conjunto dos Números Inteiros

Como de costume, usaremos o símbolo Z para representar o conjunto


dos números inteiros: . . . , −3, −2, −1, 0, 1, 2, 3, . . .. Além disso, denota-
remos por N0 = {0, 1, 2, 3, 4, . . .} o conjunto dos inteiros não negativos,
e por N = {1, 2, 3, 4, . . .} os inteiros positivos (números naturais).
O conjunto dos números inteiros é fechado para as operações de
adição e multiplicação, isto é, para quaisquer dois inteiros a, b os
números a + b e a · b são ainda inteiros. Ademais, valem as propriedades
listadas na definição abaixo.

Definição 1.1.1 (Propriedades da adição e multiplicação) Para quaisquer


inteiros a, b, c, tem-se:
Capítulo 1. Aritmética dos Inteiros

A1 ) (a + b) + c = a + (b + c); M1 ) (a · b) · c = a · (b · c);

A2 ) a + b = b + a; M2 ) a · b = b · a;

A3 ) a + 0 = a; M3 ) a · 1 = a;

A4 ) a + (−a) = 0; D) a · (b + c) = a · b + a · c.

As propriedades A1 , A2 , A3 , e A4 são chamadas de associativa,


comutativa, existência de elemento neutro e existência de inverso
aditivo, em relação a adição; do mesmo modo, relativamente à multi-
plicação, denominamos M1 , M2 e M3 . Por fim, as propriedade D, que
combina ambas operações, é conhecida por distributiva.
Em termos da adição podemos definir a subtração a − b de dois
números inteiros a e b como sendo a igualdade a + (−b) = a − b.
Note que, em virtude de A2 , tem-se que 0 + a = a + 0, −a + a =
a + (−a) e (b + c) · a = a · (b + c). Além disso, por M2 , temos 1 · a = a · 1.

Proposição 1.1.1 Para qualquer número inteiro a, tem-se que a · 0 = 0.

Prova De fato, como a · 0 + a · 0 = a · (0 + 0) = a · 0, temos, por A4 ,


a · 0 = 0. ■
Com a propriedade D podemos provar várias outras propriedades
dos números inteiros como, por exemplo, a famosa regras de sinais.

Proposição 1.1.2 (Regra de sinais) Para quaisquer dois números inteiros


a, b, tem-se que

i) a · (−b) = (−a) · b = −(a · b); e

ii) (−a) · (−b) = a · b.

2
Capítulo 1. Aritmética dos Inteiros

Prova Provaremos primeiro o item i), e depois o item ii).

i) Temos

a · (−b) + a · b = a · (−b + b)
=a·0
= 0,

o que implica em a · (−b) = −(a · b). Do mesmo modo, de (−a) ·


b + a · b = 0, podemos concluir que (−a) · b = −(a · b). Portanto
a · (−b) = (−a) · b = −(a · b);

ii) Usando o resultado anterior, temos

(−a) · (−b) + (−a) · b = (−a) · (−b + b)


= (−a) · 0
= 0,

o que nos dá (−a) · (−b) = a · b. Finalizamos assim o resultado. ■

Proposição 1.1.3 (A adição é cancelativa) Sejam a, b, c números inteiros.


Se a + c = b + c, então a = b.

Prova De fato, por A4 , adicionando −c a ambos os membros da igualda


a + c = b + c temos a = b. ■
A recíproca do resultado acima também é verdadeira, pois pelo fato
de adição em Z se bem definida, isto implica que a + c = b + c, sempre
que a = b.

3
Capítulo 1. Aritmética dos Inteiros

Exercícios

Exercício 1.1.1 Mostre que 0 e 1 são os únicos elementos neutros da


adição e multiplicação, respectivamente, em Z.

Exercício 1.1.2 Mostre que −(−a) = a.

Exercício 1.1.3 Mostre que (−1) · a = −a.

Exercício 1.1.4 Mostre que −(a + b) = −a − b.

Exercício 1.1.5 Mostre que −(b − a) = a − b.

Exercício 1.1.6 Mostre que a · (b − c) = a · b − a · c.

Exercício 1.1.7 Mostre que −a · (b − c) = a · c − a · b.

Exercício 1.1.8 Mostre que, se a + b = 0, tem-se a = −b e b = −a.

1.2 Ordenação dos números inteiros

O conjunto dos números inteiros possui um subconjunto de números


positivos, neste caso o conjunto N. Estabeleceremos agora uma relação
de ordem em Z.

Definição 1.2.1 (Ordenação)

O1 ) Tricotomia: Se a são números inteiro qualquer, vele apenas uma


das alternativas:

i) −a ∈ N;
ii) a = 0;
iii) a ∈ N

4
Capítulo 1. Aritmética dos Inteiros

O2 ) Fechamento para adição e multiplicação: Se a, b estão em N,


tem-s que

i) a + b ∈ N;
ii) a · b ∈ N.

Se indicarmos por Z− o conjunto dos números inteiros negativos e


por Z+ o conjunto dos inteiros positivos (números naturais), podemos
escrever Z = Z− ∪ {0} ∪ Z+ , para dizer que Z é a união dos inteiros
negativos, positivos e do número zero.
Estabeleceremos agora uma relação de ordem em Z, cujo objetivo é
dizer quando um inteiro é maior do que outro.

Definição 1.2.2 (Relação de ordem) Dizemos que um número inteiro a é


menor do que um inteiro b, o qual denotamos por a < b, sempre que
b − a ∈ N.

Também escrevermos, vez ou outra, b > a para dizer que b é maior


do que a.
Com a relação de ordem estabelecida na definição acima, propomos
a seguinte proposição.

Proposição 1.2.1 (Tricotomia) Para quaisquer dois números inteiros a, b,


temos: ou b < a, ou a = b ou a < b.

Prova De fato, vemos por O1 que, se a, b ∈ Z, somente uma das


seguintes alternativas acontece: ou −(b − a) ∈ N, ou a − b = 0 ou
b − a ∈ N. Se ocorrer −(b − a) = a − b ∈ N, temos b < a. Se ocorrer
a − b = 0, temos a = b. Por fim, se ocorrer b − a ∈ N, temos a < b. ■

5
Capítulo 1. Aritmética dos Inteiros

Como consequência da relação a < b, temos as seguintes proprieda-


des.

Proposição 1.2.2 (Propriedades da relação de ordem) Sejam a, b, c números


inteiros.

P1 ) Transitividade: Se a < b e b < c, então a < c;

P3 ) Monotonicidade da adição: a < b, então para qualquer inteiro


c, tem-se a + c < b + c;

P3 ) Monotonicidade da multiplicação: Se a < b, então para qual-


quer inteiro c > 0, tem-se a · c < b · c, e para qualquer inteiro
c < 0, tem-se b · c < a · c.

Prova Mostraremos P3 , e as outras duas ficam como exercício.


Digamos que a < b e c > 0. Neste caso temos que b − a e c estão
em N. Por O2 , temos (b − a)· = b · c − a · c ∈ N, logo, a · c < b · c.
Se agora, a < b e c < 0, temos que b − a e −c estão em N. Assim,
(b − a) · (−c) = a · c − b · c, o que implica em b · c < a · c. ■

Exemplo 1.2.1 Mostremos que, se a for um número inteiro não nulo,


temos que a2 > 0.

Solução Como a ̸= 0, ou a < 0 ou a > 0. Se tivermos a < 0, é óbvio


que −a > 0 e, neste caso a2 = (−a) · (−a) > (−a) · 0 = 0. Caso a > 0,
então a2 = a · a > a · 0 = 0. ■

Proposição 1.2.3 As desigualdades a < b e −b < −a são equivalentes.

Prova Mostremos que −b < −a implica em a < b e deixamos o outro


sentido como exercício. De fato, vemos que −b < −a ⇒ b · (−1) < a ·

6
Capítulo 1. Aritmética dos Inteiros

(−1) ⇒ a·(−1)−b·(−1) ∈ N ⇒ (a−b)·(−1) ∈ N ⇒ b−a ∈ N ⇒ a < b.


Proposição 1.2.4 (A multiplicação é cancelativa) Seja a, b, c números inteiro


com c ̸= 0. Se a · c = b · c, então a = b.

Prova Como c ̸= 0, ou c > 0 ou −c > 0. Consideremos o caso em


que c > 0. Se em vez de a = b, tivéssemos a < b, isto implicaria que
a · c < b · c. Já, se b < a, temos b · c < a · c. Em ambos os casos temos
um absurdo com o fato de que a · c = b · c. Logo a = b. Do mesmo
modo, conclui-se que a = b, se −c > 0. ■
Agora que temos uma garantia de que podemos cancelar fatores não
nulos na multiplicação de números inteiros, podemos provar o resultado
abaixo.

Proposição 1.2.5 Sejam a, b inteiros. Se a · b = 0, então a = 0 ou b = 0.

Prova De fato, se tivermos, digamos que a · b = 0 com a ̸= 0, então


a · b = a · 0, o que implica em b = 0. Do mesmo modo, se b ̸= 0,
temos a = 0. É claro, que se a = b = 0, temos a · b = 0. Assim, fica
demonstrado o resultado. ■
Usaremos o símbolo ⩽ para indicar que um número é menor do
que ou igual a outro, isto é, a ⩽ b, significa que a < b ou a = b. (Do
mesmo modo b ⩾ a indica que b é maior do que ou igual a a)
Sempre que tivemos um número inteiro a, sabemos que, ou −a ∈ N,
ou a = 0 ou a ∈ N. Propomos a seguintes definição.

7
Capítulo 1. Aritmética dos Inteiros

Definição 1.2.3 (Valor absoluto) Chamamos de valor absoluto de um


número inteiro a a expressão

−a, se a < 0,
|a| =
 a, se a ⩾ 0.


Usaremos o símbolo a para indicar a raiz quadrada positiva de

um inteiro a. Para expressar este fato, escrevemos a = −b quando
√ √
b < 0 ou a = b se tivermos b > 0; assim, por exemplo, 64 = 8. É
claro que, se b = 0, temos (−b)2 = b2 = 0, e deste modo, podemos

definir 0 = 0.
Perceba que, se a for um inteiro qualquer, tem-se que a ⩽ |a| (e
também −a ⩽ |a|). De fato, se a for positivo, então a = |a|. Se a for
negativo, então −a é positivo e a < −a = |a|; logo, a ⩽ |a|. Além disso,
é óbvio que |a| = a, se a = 0.

Da discussão acima não é difícil de ver que |a| = a2 , pois |a| = a
quando a ⩾ 0 e |a| = −a quando a < 0, assim, em ambos os casos
|a|2 = a2 .

Proposição 1.2.6 Para todo número inteiro a, tem-se que −|a| ⩽ a ⩽ |a|.

Prova Ora, como −a ⩽ |a| e a ⩽ |a|, temos −|a| ⩽ a e a ⩽ |a|, isto é,


−|a| ⩽ a ⩽ |a|. ■

Proposição 1.2.7 Se a e δ são números inteiros, então são equivalentes:

i) −δ ⩽ a ⩽ δ;

ii) −a ⩽ δ e a ⩽ δ;

iii) |a| ⩽ δ.

8
Capítulo 1. Aritmética dos Inteiros

Prova Ora, −δ ⩽ a ⩽ δ é equivalente a −δ ⩽ a e a ⩽ δ que, por sua


vez, equivale a −a ⩽ δ e a ⩽ δ, do qual, por fim, obtemos |a| ⩽ δ. (O
símbolo δ é uma letra grega denominada delta.) ■

Exercícios

Exercício 1.2.1 Prove que:

a) Se a < b e b < c, então a < c;

b) Se a < b, então a + c < b + c;

c) Se a < b, então −b < −a;

d) Se 1 < a, então a < a2 ;

e) Se 0 < a < 1, então a2 < a;

f) Se a ⩽ b e b ⩽ a, então a = b;

g) Se a > 0 e b > 0 (b < 0), então a · b > 0 (a · b < 0);

h) Se a < 0 e b < 0 (b > 0), então a · b > 0 (a · b < 0).

Exercício 1.2.2 Prove que, se a e b são números inteiros, então

a) |a + b| ⩽ |a| + |b|;

b) |a| − |b| ⩽ |a + b|; [Dica: escreva a = a + b − b.]

c) |a| − |b| ⩽ |a − b|;

d) |a − b| ⩽ |a| + |b|.

Exercício 1.2.3 Prove que, se a e b são números inteiros, então

|a · b| = |a| · |b|.

9
Capítulo 1. Aritmética dos Inteiros

Exercício 1.2.4 Sejam a, b ∈ Z, tais que b ̸= 0 e |b| ⩾ 1. Mostre que


|a · b| ⩾ |a|.

Exercício 1.2.5 Determine, e interprete geometricamente, todos os valores


de a tais que:

a) |a| < 5;

b) |a| > 3;

c) |a − 2| ⩽ 1;

d) |a + 4| = 8;

e) |a − 5| = 10;

10
2
Indução Matemática

Esta capítulo será dedicado aos princípios de indução matemática.

2.1 Princípio de Indução

Definição 2.1.1 Dizemos que um subconjunto S de números inteiros é


limitado inferiormente se, para todo b ∈ S, existir algum inteiro a tal
que a ⩽ b.

Admitiremos que o conjunto vazio ∅ é limitado inferiormente.


Em relação a definição acima, no caso em que a ∈ S, diremos que a
é o menor elemento de S.
É claro que, se a for o menor elemento de algum subconjunto S
de números inteiros, este deve ser único. De fato, se a ′ fosse outro
candidato a menor elemento, teríamos a ′ ⩽ a, mas como a ⩽ a ′ ,
devemos ter mesmo a = a ′ .

Axioma 2.1.1 (Princípio da boa ordenação) Todo subconjunto S números


inteiros, não vazio e limitado inferiormente, possui um menor elemento.
Capítulo 2. Indução Matemática

O princípio da boa ordenação (P.B.O) diz, em outras palavras, que


para todo b ∈ S, existe algum a ∈ S, tal que a ⩽ b.
O P.B.O desempenha papel importante neste capítulo. Mostraremos
agora uma aplicação deste na demonstração do seguinte resultado.

Proposição 2.1.1 Não existe nenhum número n natural tal que 0 < n < 1.

Prova Digamos que exista algum inteiro n tal que 0 < n < 1. Deste
modo, o conjunto S = {n; 0 < n < 1} é não vazio e limitado inferiormente
por 0, o que implica, pelo P.B.O, que S possui um menor elemento;
digamos a. Como a ∈ S, temos 0 < a < 1. Mas, deste modo, temos
0 < a2 < a < 1, o que, por sua vez, significa que a2 < a. Logo,
devemos ter a2 ∈ S, o que é um absurdo com a escolha de a como
menor elemento de S. Portanto, o resultado é verdadeiro. ■
O conjunto dos números inteiros também possui a propriedade
arquimediana. Vemos o seguinte teorema.

Teorema 2.1.1 (Propriedade arquimediana) Sejam a, b inteiros, com b ̸= 0.


Então existe algum número inteiro x tal que b · x ⩾ a.

Prova Como b ̸= 0, dividiremos a nossa demonstração em dois casos,


quando −b > 0 e quando b > 0.
Caso −b > 0, significa que −b ⩾ 1. Multiplicado essa desigualdade
por |a|, e somando −b ao resultado, obtemos b · (−|a| − 1) ⩾ −b + |a| ⩾
|a| ⩾ a. Assim, escolhendo x = −|a| − 1, temos b · x ⩾ a.
Para o caso em que b > 0, procedemos de modo análogo, e esco-
lhendo x = |a| + 1, obtemos b · x ⩾ a. ■
Outra forma de demonstrar a força do P.B.O será vista na demons-
tração do teorema abaixo.

12
Capítulo 2. Indução Matemática

Teorema 2.1.2 (Princípio de indução) Se a é um número inteiro e S um


subconjunto de Z tal que

i) a ∈ S; e

ii) sempre que n ∈ S, tem-se n + 1 ∈ S, para todo n ∈ S,

então S contém o conjunto de todos os inteiros maiores do que ou iguais


a a.

Prova Consideremos o conjunto T = {b ∈ Z; b ⩾ a}. Suponhamos, por


absurdo, que T ⊈ S. Veja que o conjunto S ′ = T \ S é não vazio, logo, é
limitado inferiormente, por a, e, pelo P.B.O, possui menor elemento;
digamos m. Ora, como m ∈ S ′ , é óbvio que m ∈
/ S, o que significa
m < a; consequentemente, m − 1 ∈ S. Agora, pelos itens i) e ii), temos
(m − 1) + 1 = m ∈ S; porém, como S ′ = T \ S, não podemos ter m ∈ S,
o que seria um absurdo com o fato de m ∈ S ′ . Portanto S contém T . ■
Uma vez que o teorema acima esta garantido, aproveitaremos para
prova abaixo um resultado que nos permitirá construir provas por
indução matemática.

Teorema 2.1.3 (Indução matemática) Se a um número inteiro e p(n) uma


proposição sobre n ∈ Z tal que

i) p(a) é verdadeira; e

ii) sempre que p(n) for verdadeira, para todo n ⩾ a, tem-se p(n + 1)
verdadeira,

então, p(n) é verdadeira, para todo n ⩾ a.

Prova Seja S o conjunto de todos os números inteiros n tais que p(n) é

13
Capítulo 2. Indução Matemática

verdadeira, isto é,

S = {n ∈ Z; p(n) é verdadeira}.

Pelo item i), temos a ∈ S, pois p(a) é verdadeira; já pelo item ii), temos
que n + 1 ∈ S, sempre que n ∈ S, desde que a veracidade de p(n)
implique na de p(n + 1). Logo, pelo teorema 2.1.2, S é o conjunto de
todos os números inteiro maiores do que ou iguais a a. Portanto, p(n)
é verdadeira, para todo n ⩾ a. ■
Aproveitaremos agora para mostrar como aplicar o teorema acima.

Exemplo 2.1.1 Mostremos que a igualdade

1 + 3 + 5 + . . . + (2n − 1) = n2

é verdadeira, para todo n ∈ N.

Solução Seja p(n) : 1 + 3 + 5 + . . . + (2n − 1) = n2 . Temos:

i) Caso base:

Vemos que p(1) é claramente verdadeira, pois 1 = 12 ;

ii) Passo indutivo:

Suponhamos a veracidade de p(n) e mostre que isto implica na


veracidade de p(n + 1). Note que (n + 1)2 = n2 + (2n + 1) = p(n) +
(2n + 1), isto é p(n + 1) é verdadeira. Portanto, p(n) é verdeira, para
todo n ∈ N. ■
Vale fazer um comentário sobre o item ii) acima. Veja que para garan-
tir p(n+1), supondo p(n) verdadeira, o que fizemos foi simplesmente adi-
cionar (2n+1) a ambos os membros da igualdade 1+3+5+. . .+(2n−1) =
n2 ; o que é comum quando tratamos de igualdade.

14
Capítulo 2. Indução Matemática

Exemplo 2.1.2 (Desigualdade de Bernoulli). Mostremos que é válida a


desigualdade (1 + h)n ⩾ 1 + h · n, sempre que h > −1 e n ∈ N0 .

Solução Seja p(n) : (1 + h)n ⩾ 1 + h · n. Temos:

i) Caso base:

Vemos que p(0) é verdadeira, pois (1 + h)0 ⩾ 1 + h · 0 = 1;

ii) Passo indutivo:

Suponhamos p(n) e mostremos p(n + 1). Veja que

(1 + h)n+1 = (1 + h)n · (1 + h)
= (1 + h)n + h · (1 + h)n
⩾ (1 + h · n) + h · (1 + h · n)
⩾ 1 + h · (n + 1) + h2 · n
⩾ 1 + h · (n + 1),

o que garante a veracidade de p(n + 1). Portanto, p(n) é verdadeira,


para todo n ∈ N0 . ■
O teorema 2.1.3 tem uma versão mais completa que apresentaremos
agora.

Teorema 2.1.4 (Indução matemática completa) Se a um número inteiro e


p(n) uma proposição sobre n ∈ Z tal que

i) p(a) é verdadeira; e

ii) sempre que p(a), p(a + 1), . . . , p(n) forem verdadeiras, para todo
n, tem-se p(n + 1) verdadeira,

então, p(n) é verdadeira, para todo n ⩾ a.

15
Capítulo 2. Indução Matemática

Prova Seja S o conjunto de todos os números inteiros tais que p(n) é


verdadeira, isto é,

S = {n ∈ Z; p(n) é verdadeira}.

Suponhamos, por absurdo, que S ̸= T = {b ∈ Z; b ⩾ a}, e seja S ′ = T/S.


Ora, S ′ é não vazio e limitado inferiormente, por a, assim, pelo P.B.O,
possui um menor elemento; digamos m.
Pelo item i) a ∈ S, já p(a) é verdadeira, o que nos diz que m > a.
Como m é o menor inteiro que não pertence a S, isto é, (m − 1) ∈ S, e
assim p(a), p(a + 1), . . . , p(m − 1) são todas verdadeiras. Porém, pelo
item ii), p(m) é verdadeira, logo, m ∈ S; o que é uma contradição, pois
m ∈ S ′.
Portanto, S = {b ∈ Z; b ⩾ a}, e p(n) é verdadeira, para todo n ⩽ a,
como queríamos. ■

Exemplo 2.1.3 Mostremos que an = 2n − 1, para todo n ⩾ 1, desde que


a1 = 1, a2 = 3 e an = 2an−1 + an−2 , para n ⩾ 3.

Solução Seja p(n) : an = 2n − 1. Temos:

i) Caso base:

Vemos que p(3) é verdadeira, pois a3 = 2(3) − 1 = 5 e a3 =


2a2 − a1 = 2(3) − 1 = 5;

ii) Passo indutivo:

Suponhamos p(3), p(4), . . . , p(n), para todo n ⩾ 3, e mostremos


p(n + 1).

16
Capítulo 2. Indução Matemática

Veja que

an+1 = 2an − an−1


= 2(2n − 1) − (2n − 3)
= 4n − 2 − 2n + 3
= 2n + 1
= 2n + 2 − 1
= 2(n + 1) − 1,

isto, é p(n + 1) é verdadeira. Portanto, p(n) é verdadeira, para todo


n⩾1 ■

Exercícios

Exercício 2.1.1 Use indução matemática para verificar as igualdades


abaixo.
n(n+1)(2n+1)
a) 12 + 22 + 32 + . . . + n2 = 6 , para todo n ⩾ 1;
h i2
n(n+1)
b) 13 + 23 + 33 + . . . + n3 = 2 , para todo n ⩾ 1;

(2n−1)n(2n+1)
c) 12 + 32 + 53 + . . . + (2n − 1)2 = 3 , para todo n ⩾ 1;

d) 13 + 33 + 53 + . . . + (2n − 1)3 = n2 (2n2 − 1), para todo n ⩾ 1;


n(n+1)(n+2)
e) 1 · 2 + 2 · 3 + . . . + n(n + 1) = 3 , para todo n ⩾ 1.

Exercício 2.1.2 Use indução matemática para verificar as desigualdades


abaixo.

a) 2n > 2n−1 , para todo n ⩾ 1;

17
Capítulo 2. Indução Matemática

b) 2n > n2 , para todo n ⩾ 5;

1 1 1 1
c) 1 + 22
+ 32
+ ... + n2
⩽2− n, para todo n ⩾ 1.

Exercício 2.1.3 Use indução completa para verificar que

an − 1 = (a − 1)(an−1 + an−2 + . . . + a + 1).

[Dica: use an+1 = (a + 1)(an − 1) − a(an−1 − 1).]

Exercício 2.1.4 Considere que, para todo n ∈ N, dado a1 , a igualdade


an+1 − an = d é uma progressão aritmética. Mostre que

a) an = a1 + (n − 1)d;
(a1 +an )n
b) Sn = 2 , desde que Sn = a1 + a2 + . . . + an .

Exercício 2.1.5 Considere que, para todo n ∈ N, dado a1 , a igualdade


an+1
an = q é uma progressão geométrica. Mostre que

a) an = a1 qn−1 ;
n
b) Sn = a1 qq−1
−1
, desde que Sn = a1 + a2 + . . . + an .

Exercício 2.1.6 Mostre que an = 5 · 2n + 1, para todo n ⩾ 1, desde que


a1 = 11, a2 = 21 e an = 3an−1 − 2an−2 , para n ⩾ 3.

2.2 Binômio de Newton

A indução matemática nos dá uma maneira de entender o significado


preciso das reticências numa soma do tipo a1 + a2 + . . . + an , uma vez
que não somos capazes de somar mais do que dois números ao mesmo
tempo.

18
Capítulo 2. Indução Matemática

Podemos interpretar Sn = a1 + a2 + . . . + an como uma soma uma


sequência do tipo

S1 = a1 ,
S2 = S1 + a2 ,
S3 = S2 + a3 ,
..
.
Sn = Sn−1 + an ,

em que cada termo é obtido a partir de um ou mais termos anteriores.


Em outras palavras, se pusermos p(a) : S1 , p(a + 1) : S2 , . . . , p(n) : Sn ,
temos p(n + 1) : Sn+1 .

Pn
Definição 2.2.1 (Somatório) O somatório i=1 ai = Sn , em que Sn =
a1 + a2 + . . . + an , é definido por
P1
i) i=1 ai = a1 ; e
Pn+1 Pn
ii) i=1 ai = i=1 ai + an+1 .

Vemos na definição acima que as reticências ficam claras, uma vez


que tal definição é garantida por indução matemática.

Qn
Definição 2.2.2 (Produtório) O produtório i=1 ai = Pn , em que
Pn = a1 · a2 · . . . · an , é definido por
Q1
i) i=1 ai = a1 ; e
Qn+1 Qn
ii) i=1 ai = i=1 ai · an+1 .

19
Capítulo 2. Indução Matemática

Definição 2.2.3 (Fatorial) O fatorial n! = n · (n − 1) · . . . · 2 · 1 é definido


por

i) 1! = 1; e

ii) (n + 1)! = (n + 1) · n!.

A afim de evitarmos complicações futuras, convencionamos 0! = 1.


Consideremos a representação numérica abaixo, conhecida como
triângulo de Pascal.

n
0 1
1 1 1
2 1 2 1
3 1 3 3 1
4 1 4 6 4 1
5 1 5 10 10 5 1
6 1 6 15 20 15 6 1
0 1 2 3 4 5 6
i

n

Representaremos cada um dos números acima pelo símbolo i que,
por sua vez, significa o elemento que está na linha (nível) n e coluna i
do triângulo; assim, por exemplo, 42 = 6.


Definição 2.2.4 (Coeficiente binomial) Sejam n e i elementos em N0 .


Chamamos de coeficiente binomial os números ni do triângulo de


Pascal.

n n n
  
Colocaremos 0 = n = 1, para n ⩾ i, e i = 0, para n < i.

20
Capítulo 2. Indução Matemática

Uma olhada atenta no triângulo de Pascal nos mostra, por exemplo,


que 53 = 43 + 42 ; em outras palavras, o elemento que está na linha
  

5 e coluna 3 é obtido da soma dos elementos que estão na linha 4 e


coluna 3 e 2, respectivamente.

 P
Proposição 2.2.1 (Relação de Stifel) Seja ni = m m

k=1 i−1 , para m < n,
com n 0
 
0 = 1, para todo n ∈ N, e i = 0, para i > 0. Então
     
n n−1 n−1
= + .
i i i−1

Prova Provaremos por indução sobre i. Se i = 1, podemos checar que


n n n−1
+ n−1
   
1 = n, o que satisfaz 1 = 1 0 . Suponhamos agora que,
′
para todo i ′ < i, tem-se ni ′ . Deste modo,
  X m  
n m
=
i i−1
k=1
Xm 
m−1
 X m 
m−1

= +
i−1 i−2
k=1 k=1
   
n−1 n−1
= + ,
i i−1

o que conclui o nosso resultado. ■


No nosso próximo resultado mostraremos como determinas os nú-
meros ni através de uma fórmula.


Proposição 2.2.2 Sejam i, n ∈ N, com 1 ⩽ i ⩽ n. Então,


 
n n(n − 1) . . . (n − (i − 1))
= .
i i!

21
Capítulo 2. Indução Matemática

Prova Usaremos indução sobre n. Se n = 1 não há o que discutir.


Suponhamos a validade desta fórmula para i, tal que 1 ⩽ i ⩽ n, para
n ∈ N. Pela relação de Stifel, temos
     
n+1 n n
= +
i i i−1
n(n − 1) . . . (n − (i − 1)) n(n − 1) . . . (n − (i − 2))
= +
i! (i − 1)!
n(n − 1) . . . (n − (i − 1)) n(n − 1) . . . (n − (i − 2))i
= +
i! i!
n(n − 1) . . . (n − (i − 2))(n − (i − 1) + i)
=
i!
(n + 1)n(n − 1) . . . (n − (i − 2))
= ,
i!

o que prova nossa fórmula. ■


Como consequência do resultado acima, se multiplicarmos e dividir-
mos ni por (n − i)!, obtemos


 
n n!
= .
i i!(n − i)!

Teorema 2.2.1 (Binômio de Newton) Sejam a, b números reais quaisquer


e n ∈ N. Então,
     
n nn n−1 n n−2 2 n
(a+b) = a + a b+ a b +. . .+ abn−1 +bn .
1 2 n−1

Prova Provaremos por indução. Primeiramente, note que para n = 1 e


n = 2 o resultado é verdadeiro. Suponhamos que a fórmula do binômio
seja válida para todo n ⩾ 2. Consideremos (a+b)n+1 = (a+b)(a+b)n ,

22
Capítulo 2. Indução Matemática

temos

(a + b)n+1 = (a + b)(a + b)n


   
n+1 n n+1
= an+1 + a b + ... + abn + bn+1 ,
1 n

n+1 n n
  
em que i = i + i−1 . Portanto, o teorema fica concluído. ■

Exercícios

Exercício 2.2.1 Sejam a, b números reais e m, n ∈ N0 . Mostre, por


indução sobre n, que

a) am · an = am+n ;

b) (am )n = amn ;

c) (ab)n = an · bn ,

desde que, para todo a ̸= 0, a potência an esteja definida por an+1 =


an a, com a0 = 1 e a1 = a.

Exercício 2.2.2 Determine uma fórmula para 1 · 1! + 2 · 2! + . . . + n · n!.


[Dica: use o fato de que n · n! = (n + 1)! − n!.]

Exercício 2.2.3 Mostre que, para todo n ∈ N e todo i, com 0 ⩽ i ⩽ n,


tem-se    
n n
= .
i n−i
Exercício 2.2.4 Mostre que 2n = n n n
  
0 + 1 + ... + n .

Exercício 2.2.5 Mostre que, para todo n ⩾ 4, tem-se 2n > n!

Exercício 2.2.6 Mostre que, para todo n ⩾ 4, tem-se n! > n2 .

23
Capítulo 2. Indução Matemática

Exercício 2.2.7 Mostre que, para todo n ⩾ 1, tem-se


       
n n n n
n2n−1 = +2 +3 + ... + n .
1 2 3 n

24
3
Divisão Euclidiana

Este capítulo será dedicado ao estuda da operação de divisão dos


números inteiros e suas consequências.

3.1 Divisibilidade

Definição 3.1.1 (Relação de divisibilidade) Sejam a, b números inteiros,


com b ̸= 0. Dizemos que b divide a, o qual denotamos por b | a, se
existir algum inteiro c tal que a = bc.

Pela definição acima, vemos que 2 divide 10, pois 10 = 2 · 5. Do


mesmo modo, 5 divide 10.
Quando b não divide a escreveremos b ∤ a, para indicar que não
existe c ∈ Z tal que a = bc.
Se b divide a, diremos que b é um divisor de a, ou que b é um
múltiplo de a.

Observação 3.1.1 Cuidado com as escritas b | a e b/a. A primeira é uma


relação entre dois números inteiros, enquanto que a segunda significa o
quociente entre b e a.
Capítulo 3. Divisão Euclidiana

Proposição 3.1.1 Sejam a, b inteiros, com b não nulo. Se b | a, isto é


a = bc, então c é único.

Prova De fato, se existir c ′ tal que a = bc ′ = bc, temos b(c − c ′ ) = 0.


Como b ̸= 0, pela lei do cancelamento da multiplicação em Z, só
podemos ter c = c ′ . ■

Proposição 3.1.2 Se b | a, então b | ac.

Prova De fato, se tivermos b | a, então existe q ∈ Z tal que a = bq.


Multiplicando esta igualdade por c, temos ac = bq ′ , logo b | ac. ■

Proposição 3.1.3 Se b divide a e a > 0, então b ⩽ a.

Prova Como b | a, existe c tal que a = bc. Se, por ventura, tivermos
a < b, isto é, 0 < a < b, fica claro que 0 < c. Assim, 0 < ac < bc = a,
o que implica em 0 < c < 1; porém, isso é um absurdo, segundo a
proposição 2.1.1. Portanto, o resultado é verdadeiro. ■

Proposição 3.1.4 Se b | 1, então b = ±1.

Prova De fato, se tivemos b | 1, então existe c ∈ Z tal que 1 = bc; o


que implica em b = c = 1 ou b = c = −1. Portanto, b = ±1. ■

Proposição 3.1.5 Se b | a e a | b, então b = ±a.

Prova Ora, se tivemos b | a e a | b, então existem c, c ′ ∈ Z tais que


a = bc e b = ac ′ . Deste modo, a = a(cc ′ ), o que implica em cc ′ = 1.
Como 1 = cc ′ , vemos que c | 1, o que significa c = c ′ = ±1, e portanto,
b = ±a. ■

26
Capítulo 3. Divisão Euclidiana

Proposição 3.1.6 Se b | a e a ̸= 0, então |b| ⩽ |a|.

Prova De fato, se b | a e a ̸=, existe c tal que a = bc, com c ̸= 0; logo,


|a| = |b||c|. Como |c| ̸= 0, isto significa que |c| ⩾ 1, logo, |a| = |b||c| ⩾ |b|,
e portanto, |b| ⩽ |a|. ■

Proposição 3.1.7 Sejam x, y números inteiros quaisquer. Se b | a e b | c,


então b | (ax + cy).

Prova De fato, se b | a e b | c, exitem s, t ∈ Z tais que a = bs e c = bt.


Multiplicando a primeira dessas igualdades por x e a segunda por y e
somando os resultados, obtemos

ax + cy = b(sx + ty),

o que implica em b | (ax + cy). ■


A escrita ax + by é chamada de combinação linear de a, b, x, y.
Provaremos agora, por indução matemática, três resultados interes-
santes.

Proposição 3.1.8 Sejam a, b inteiros quaisquer e n ∈ N. Então, a − b


divide an − bn .

Prova Se n = 1, não há dúvidas, pois a − b | a1 − b1 . Suponhamos


a − b | an − bn . Assim,

an+1 − bn+1 = aan − bbn


= aan − ban + ban − bbn
= (a − b)an + b(an − bn ).

27
Capítulo 3. Divisão Euclidiana

Como a−b | a−b e a−b | an −bn , vemos que a−b | an+1 −bn+1 ,
o que garante o nosso resultado. ■

Exemplo 3.1.1 Escrevendo 3 = 5 − 2 vemos que 3 divide 51 0 − 21 0.

Proposição 3.1.9 Sejam a, b inteiros quaisquer e n ∈ N. Então, a + b


divide a2n − b2n .

Prova Se n = 1 não o que dizer, pois a + b | a2 − b2 = (a − b)(a + b).


Suponhamos que a + b | a2n − b2n . Assim,

a2(n+1) − b2(n+1) = a2 a2n − b2 b2n


= a2 a2n − b2 a2n + b2 a2n − b2 b2n
= (a2 − b2 )a2n + b2 (a2n − b2n ).

Como a + b | a2 − b2 e a + b | a2n − b2n , vemos que a + b |


a2(n+1) − b2(n+1) , o que garante o nosso resultado. ■

Exemplo 3.1.2 Escrevendo 16 = 25 − 9, vemos que 16 divide 256 − 166 .

Proposição 3.1.10 Sejam a, b inteiros e n ∈ N0 . Então, a + b divide


a2n+1 + b2n+1 .

Prova Se n = 0 não há o que dizer, pois a + b | a1 + b1 . Suponhamos


que a + b | a2n+1 + b2n+1 . Assim,

a2(n+1)+1 + b2(n+1)+1 = a2 a2n+1 + b2 b2n+1


= a2 a2n+1 − b2 a2n+1 + b2 a2n+1 + b2 b2n+1
= (a2 − b2 )a2n+1 + b2 (a2n+1 + b2n+1 ).

Como a + b | a2 − b2 e a + b | a2n+1 + b2n+1 , vemos que a + b |


a2(n+1)+1 + b2(n+1)+1 , o que garante o nosso resultado. ■

28
Capítulo 3. Divisão Euclidiana

Exemplo 3.1.3 Escrevendo 7 = 5 + 2, vemos que 7 divide 57 + 27 .

Como nem sempre é possível fazer a divisão entre inteiros, admi-


tiremos que podemos ainda proceder com a divisão se aceitarmos a
possibilidade de sobrar algum resto. Propomos o seguinte teorema.

Teorema 3.1.1 (Algoritmo da divisão) Sejam a, b números inteiros, com


b ̸= 0. Então, existem únicos inteiros q e r tais que

a = bq + r com 0 ⩽ r < |b|.

Prova Comecemos considerando o conjunto

S = {a − bx; x ∈ Z}

composto de inteiros não negativos. S é não vazio, pois se, a ⩾ 0, temos


a = a − b · 0 ∈ S. Se, por sua vez, −a > 0, tomamos −b > 0, o que nos
dá (−b)(−a) ⩾ −a, isto é, a − b · (−a) ∈ S.

i) Existência:

O conjunto S ′ de todos os inteiros não negativos de S é limitado


inferiormente, por 0, logo, pelo P.B.O, possui um menor elemento,
digamos r. Assim, existe q ∈ Z tal que r = a − bq ∈ S ′ , com 0 ⩽ r.
Devemos ter r < |b|, pois caso contrário, se tivermos |b| ⩽ r, como
|b| ⩾ 1 há algum c ∈ S ′ tal que r = c + |b|, isto é, c = r − |b|. Deste
modo 0 ⩽ c < r, o que é um absurdo com o fato de r ser o menor
elemento de S ′ . Logo, temos mesmo r < |b|.
Se b > 0, escolhemos q tal que a = bq + r, com 0 ⩽ r < b. Se
−b > 0 escolhemos −q tal que a = bq + r; com 0 ⩽ r ⩽ −b. Em ambos
os casos a = bq + r, com 0 ⩽ r < |b|. Concluímos assim a existência de
q e r.

29
Capítulo 3. Divisão Euclidiana

ii) Unicidade:

Suponhamos a = bq + r e a = bq ′ + r ′ , com 0 ⩽ r ′ , r < |b|. Assim,


b(q − q ′ ) = −(r − r ′ ), tal que |b| divide |r − r ′ |. Como r, r ′ < |b|, é
claro que, |r − r ′ | < |b|; logo, só podemos ter |r − r ′ | = 0, isto é, r = r ′ ,
o que, por sua vez, implica em q = q ′ .
Concluímos assim que q e r são únicos. Portanto, o teorema é
verdadeiro. ■
Aproveitemos para ilustremos o algoritmo da divisão no caso em
que b < 0. Para isso, escolhamos b = −3 e a = 1, 5, e −9:

1 = (−3) · 0 + 1
5 = (−3) · (−1) + 2
−9 = (−3) · 4 + 3

Na fórmula a = bq + r, o número q é chamado de quociente e r é


dito ser o resto.
Quando dividimos um número por 2, segundo o algoritmo da divisão,
os possíveis restos são 0 ou 1, uma vez que 0 ⩽ 2 < |b|. Para ressaltar a
divisão por 2 propomos a seguinte definição.

Definição 3.1.2 (Paridade) Seja a um número inteiro qualquer. Dizemos


que a é par, se existe algum k ∈ Z tal que a = 2k. Se para algum k ∈ Z
tivemos a = 2k + 1, dizemos que a é ímpar.

Como exemplo, o número 6 = 2 · 3 é par, e 7 = 2 · 3 + 1 é ímpar.

Exemplo 3.1.4 Mostremos que o quadrado de um número par é ainda


par e o de um ímpar é ainda ímpar.

30
Capítulo 3. Divisão Euclidiana

Solução Digamos que a ∈ Z seja par, isto é a = 2k, logo, a2 = (2k)2 =


4k2 = 2(2k2 ).
Se a ∈ Z for ímpar, isto é, a = 2k + 1, temos a2 = (2k + 1)2 =
2(2k2 + 2k) + 1. ■
O algoritmo da divisão nos garante que se a ∈ Z for um inteiro
qualquer, podemos escrever a = 4k + r. Como r = 0, 1, 2 ou 3, temos:
a = 4k, 4k + 1, 4k + 2 ou 4k + 3.

Exemplo 3.1.5 Mostremos que o quadrado de qualquer número ímpar é


da forma 8k + 1.

Solução Como um número ímpar pode ser escrito como 4k + 1 ou 4k + 3,


vemos que

(4k + 1)2 = 8(2k2 + k) + 1


(4k + 3)2 = 8(2k2 + 3k + 2) + 1,

como queríamos. ■

Exercícios

Exercício 3.1.1 Mostre que, se b | a e a | c, então b | c.

Exercício 3.1.2 Mostre que, se b | a e d | c, então bd | ac.

Exercício 3.1.3 Mostre que, se b | a e b | a + c, então b | c.

Exercício 3.1.4 Mostre que, se b | a, então −b | a.

Exercício 3.1.5 Mostre que, se b | a e b ∤ a + c, então b ∤ c.

Exercício 3.1.6 Mostre que, para todo a ∈ Z, tem-se que 1 | a.

Exercício 3.1.7 Mostre que, se a ̸= 0, então a | a e a | 0.

31
Capítulo 3. Divisão Euclidiana

Exercício 3.1.8 Mostre que, se b | a, então b | ac.

Exercício 3.1.9 Mostre que, se b | a, então b | −a, −b | a e −b | −a.

Exercício 3.1.10 Mostre que, se a for um inteiro qualquer, então a, a + 2


e a + 3 são divisíveis por 3.

Exercício 3.1.11 Mostre que, i! divide o produto de i números naturais


consecutivos.

Exercício 3.1.12 Mostre que 6 divide n(n + 1)(n + 2), para todo n ∈ N.

Exercício 3.1.13 Mostre, por indução, que para todo n ∈ N0 :

a) 8 | 32n + 7; b) 9 | 10n + 3 · 4n+2 + 5.

Exercício 3.1.14 Mostre, sem usar indução, que para todo n ∈ N0 :

a) 10 | 11n − 1;

b) 13 | 92n − 24n;

c) 17 | 102n+1 + 72n+1 .

Exercício 3.1.15 Determine a ∈ N tal que

a) a − 2 | a3 + 4; b) a + 2 | a4 + 2a3 + a2 + 1.

Exercício 3.1.16 Mostre que, para todo a ∈ N:

a) 2 | a2 − a;

b) 3 | a3 − a;

c) 5 | a5 − a;

d) 7 | a7 − a;

32
Capítulo 3. Divisão Euclidiana

Exercício 3.1.17 Mostre, por indução, que o resto da divisão de 10n por
9 é 1, para todo n ∈ N.

Exercício 3.1.18 Mostre que, se a for um inteiro qualquer, então a2 =


3k ou 3k + 1.

Exercício 3.1.19 Mostre que, se a, b são inteiros quaisquer, então a e


a + 2b tem a mesma paridade.

Exercício 3.1.20 Mostre que, se a for ímpar, então 24 | a(a2 − 1).

3.2 O Máximo Divisor Comum

Definição 3.2.1 (Divisor comum) Sejam a, b, d números inteiros, com


b ̸= 0. Se d | a e d | b dizemos que d é um divisor comum de a e b.

O número 2 é um divisor comum de 6 e 12. Do mesmo modo 3, −3


e −2.
Para nossos estudos estamos particularmente interessados nos divi-
sores positivos. Propomos a seguinte definição.

Definição 3.2.2 (Máximo divisor comum) Sejam a, b inteiros, não ambos


nulos. Chamamos de máximo divisor comum de a e b, denotado por
(a, b), o inteiro positivo d tal que

i) d | a e d | b; e

ii) se c | a e c | b, então c | d.

Encurtaremos a palavra máximo divisor comum pela abreviação


mdc.

33
Capítulo 3. Divisão Euclidiana

Exemplo 3.2.1 Os divisores comuns de 24 e 84 são ±1, ±2, ±3, ±4, ±6 e


±12. Assim, (24, 84) = 12 é o mdc de 24 e 84.

Mostraremos agora, no teorema abaixo, que para quaisquer dois


números inteiros, não ambos nulos, o mdc sempre existe.

Teorema 3.2.1 (Identidade de Bézout) Sejam a, b números inteiros, não


ambos nulos. Então, existe e é único o mdc de a, b tal que (a, b) =
ax + by, para x, y ∈ Z.

Prova Consideremos I(a, b) = {ax + by; x, y ∈ Z} o conjunto de todas


as combinações lineares de a, b. Vemos que I(a, b) é não vazio e contém
inteiros positivos, pois a2 + b2 ∈ I(a, b).

i) Existência:

Consideremos S o subconjunto de I(a, b) composto de números


positivos. S é limitado inferiormente, por 0, logo, pelo P.B.O, possui
um menor elemento; digamos d. Além disso, pelo fato de d ser elemento
de I(a, b), exitem inteiros x, y tais que d = ax + by.
Mostraremos que todo elemento em I(a, b) é divisível por d. De
fato, se d ∤ a, pelo algoritmo da divisão, temos que existem únicos
inteiros q, r tais que a = dq + r, com 0 ⩽ r < d. Assim,

r = a − dq = a − (ax + by)q = a(1 − qx) + b(−qx),

o que nos mostra que r ∈ I(a, b), pois é uma combinação linear de a, b.
Como 0 ⩽ r < d, vemos que r não pode ser positivo, pois teríamos
r ∈ S, o que seria uma contradição com o fato de d ser o menor membro
positivo de S; logo, r = 0. Deste modo, vemos que a = dq, o que
implica em d | a. Com argumento análogo, podemos concluir que d | b.

34
Capítulo 3. Divisão Euclidiana

Em outras palavras, d é divisor comum de a, b, o que significa que


d | (ax + by).
Seja agora, c, outro divisor comum de a, b, vemos que c | (ax + by),
isto é, c divide todas as combinações lineares de a, b; logo c | d.
Portanto, d = (a, b) é mdc de a, b tal que (a, b) = ax + by.

ii) Unicidade:

Se d ′ for também mdc de a, b, então, pelo item ii) da definição


acima, d ′ | d e d | d ′ . Como d, d ′ > 0, só podemo ter d = d ′ . ■
O teorema acima, embora garanta a existência do mdc, não mostra
uma forma de calculá-lo.
É imediato que (a, b) = (b, a), isto é, o mdc é comutativo. Também
não é difícil de verificar que (0, a) = |a|, (1, a) = 1 e (a, a) = |a|.
(Verifique.)

Exemplo 3.2.2 O mdc entre 12 e 28 é 4, isto é, (12, 28) = 4, que, por


sua vez, pode ser escrito como

4 = 28(1) + 12(−2).

Porém, também podemos escrever 8 = 28(−1) + 12(3), o significa que,


o mdc não é a única combinação linear de 12, 28.

Teorema 3.2.2 (Caracterização do mdc) Sejam a, b inteiros não ambos


nulos. Se (a, b) é o mdc de a, b, então

i) (a, b) = ax+by é a menor combinação linear positiva em I(a, b) =


{ax + by; x, y ∈ Z};

ii) (a, b) é o divisor positivo de a e b que é divisível por todo c tal


que c | a e c | b.

35
Capítulo 3. Divisão Euclidiana

Prova Segue-se do teorema 3.2.1. ■


Sempre que tivermos dois inteiros a, b tais que (a, b) = 1, daremos
um nome especial a eles.

Definição 3.2.3 (Primos entre si) Dois números inteiros a, b são ditos
primos entre si se tivermos (a, b) = 1.

Algumas vezes podemos usar as expressões coprimos, primo de ou


primo com para dizer que a e b são primos entre si.

Exemplo 3.2.3 Os número 5 e 9 são primos entre si, isto é, (2, 5) = 1.

Teorema 3.2.3 Sejam a, b inteiros, não ambos nulos. Então, a e b


são primos entre si se, e somente se, existem inteiros x, y tais que
1 = ax + by.

Prova Se tivermos (a, b) = 1, pelo teorema anterior, existem inteiros


x, y tais que 1 = ax + by.
Suponhamos agora, tivermos que existem x, y inteiros tais que
1 = ax + by, e que d = (a, b). Como d é divisor comum de a e b,
sabemos que d | (ax + by), logo, d | 1. Como d é positivo, só podemos
ter d = 1. Assim, (a, b) = 1, o que significa que a e b são primos entre
si. ■
O resultado abaixo será de grande utilidade no procedimento que
veremos a seguir.

Proposição 3.2.1 Sejam a, b, n inteiros. Então (a, b) = (a, b − an).

Prova Sejam d = (a, b) e d ′ = (a, b − an). Mostraremos que d | d ′ e


d ′ | d.

36
Capítulo 3. Divisão Euclidiana

Ora, como d é divisor comum de a e b, é claro que, d | b e d | −an,


logo, d | b − an, o que implica que d também é divisor comum de a e
b − an, isto é, d | d ′ .
Agora, por ser d ′ um divisor comum de a e b, vemos que d | a e
d | b = b − an + an; isto é, d ′ é divisor comum de a e b, o implica em
d ′ | d.
Uma vez que temos d | d ′ e d ′ | d, só podemos concluir que d = d ′ ;
em outras palavras (a, b) = (a, b − an) ■

Exemplo 3.2.4 Calculemos (n, 2n + 1), para n ∈ N.

Solução Temos (n, 2n + 1) = (n, 2n + 1 − 2n) = (n, 1) = 1. ■

Algoritmo de Euclides

Mostraremos agora como calcular o mdc dados dois inteiros a e b.


Para isso, utilizaremos um dispositivo prático chamado Algoritmo de
Euclides.
Uma vez que(|a|, |b|) = (a, b), não há prejuízo algum supormos
a ⩾ b > 0; assim, pelo Algoritmo da Divisão, podemos escrever

a = bq1 + r1 com 0 ⩽ r1 < b.

Há duas possibilidades sobre r1 : ou r1 | b ou r1 ∤ b. Se r1 | b, então


r1 = (b, r1 ). Logo, pela proposição acima

r1 = (b, a − bq1 ) = (b, a) = (a, b),

o acaba o algoritmo. Caso tenhamos que r1 ∤ b, podemos dividir b por

37
Capítulo 3. Divisão Euclidiana

r1 , obtendo

b = r1 q2 + r2 com 0 ⩽ r2 < r1 < b.

Novamente temos duas possibilidade sobre r2 e r1 : ou r2 | r1 ou r2 | r1 .


Se r2 | r1 , então r2 = (r1 , r2 ), e novamente pela proposição acima

r2 = (r1 , b − r1 q2 ) = (r1 , b) = (a − bq1 , b) = (a, b),

o que acaba o algoritmo. Caso tenhamos que r2 ∤ r1 , podemos dividir


r1 por r2 , obtendo

r1 = r2 q3 + r3 com 0 ⩽ r3 < r2 < r1 < b.

Continuamos o processo acima até obtermos o resto rn+1 = 0, o que


acontecerá, pois, segundo o Algoritmo da Divisão, temos a = bq + r,
com 0 ⩽ r < b. Neste caso, o último resto não nulo, rn , será o mdc de
a e b, isto é, rn = (a, b).

Exemplo 3.2.5 Determinemos (8249, 7519) e encontremos x, y tais que


(8249, 7519) = 8249x + 7519y.

Solução Usando o Algoritmo de Euclides sucessivamente, obtemos

8249 = 7519(1) + 730


7519 = 730(10) + 219
730 = 219(3) + 73
219 = 73(3) + 0,

logo, (8249, 7519) = 19.


Encontremos agora x, y tais que (8249, 7519) = 8249x + 7519y.

38
Capítulo 3. Divisão Euclidiana

Temos

73 = 730 − 219(3)
= 730 − (7519 − 730 · 10)(3)
= 7519(−3) + 730(31)
= 7519(−3) + (8249 − 7519)(31)
= 8249(31) + 7519(−34),

logo, (8249, 7519) = 8249(31) + 7519(−34) ■

Exercícios

Exercício 3.2.1 Determine

a) (227, 143); c) (1479, 272);

b) (657, 306); d) (3256, 2746).

Exercício 3.2.2 Determine x, y tais que

a) (1218, 648) = 1218x + 648y; c) (138, 24) = 138x + 24y;

b) (874, 551) = 874x + 551y; d) (272, 119) = 272x + 119y.

Exercício 3.2.3 Seja (a, b) = 1. Mostre que

a) (a + b, a − b) = 1 ou 2;

b) (a + b, a2 + b2 ) = 1 ou 2;

c) (2a + b, a + 2b) = 1 ou 3;

d) (a + b, a2 − ab + b2 ) = 1 ou 3;

39
Capítulo 3. Divisão Euclidiana

Exercício 3.2.4 Mostre que, para n ∈ N, tem-se que

a) (n + 1, n2 + n + 1) = 1;

b) (2n + 1, 9n + 4) = 1;

c) (n! + 1, (n + 1)! + 1) = 1.

Exercício 3.2.5 Mostre que (a, b) = (a, b + an), para n ∈ N.

Exercício 3.2.6 Sejam a, b, q, r inteiros, tais que a = bq + r, com


0 ⩽ r < b. Mostre que (a, b) = (b, r).

Exercício 3.2.7 Mostre que (a, b, c) = ((a, b), c). (Generalize este resul-
tado.)

3.3 Consequências do Máximo Divisor Comum

Teorema 3.3.1 (Lema de Euclides) Sejam a, b, c números inteiros. Se


b | ac e (a, b) = 1, então b | c.

Prova Se tivemos b | ac e (a, b) = 1, existem x, y, q ∈ Z tais que


ac = bq e 1 = ax + by. Multiplicando 1 = ax + by por c, obtemos

c = c · 1 = c(ax + by) = (ac)x + b(cy) = b(qx + cy),

do qual segue-se b | c, como queríamos. ■

Exemplo 3.3.1 Observe que, se tivermos apenas que b | ac, não temos
garantia de que b | c; ilustremos isso com: 9 | 6 · 12, mas 9 ∤ 12, pois
(9, 6) ̸= 1.

40
Capítulo 3. Divisão Euclidiana

Teorema 3.3.2 Sejam a, b, c números inteiros. Então (a, b) = (a, c) = 1


se, e somente se, (a, bc) = 1.

Prova Suponhamos, primeiramente, que (a, b) = (a, c) = 1. Sabemos


que existem inteiros x, y, x ′ , y ′ tais que (a, b) = ax + by e (a, c) =
ax ′ + cy ′ . Como 1 = ax + by = ax ′ + cy ′ , temos

1 = 1 · 1 = (ax + by)(ax ′ + cy ′ )
= a(axx ′ + cxy ′ + byx ′ ) + bc(yy ′ ),

do qual segue-se (a, bc) = 1.


Agora, suponhamos (a, bc) = 1. Pondo (a, b) = d, vemos que d | a
e d | b, logo d | a e d | bc. Deste modo (a, bc) ⩾ d, o que força d = 1,
isto é (a, b) = 1. Analogamente concluímos que (a, c) = 1, o que prova
o teorema. ■

Teorema 3.3.3 Sejam a, b, c inteiros. Se a | c e b | c, com (a, b) = 1,


então ab | c.

Prova Se tivemos a | c e b | c, com (a, b) = 1, então existem inteiros


x, y, q, q ′ tais que c = aq = bq ′ e 1 = ax + by. Como

c = c · 1 = c(ax + by) = ab(q ′ x + qy),

segue-se ab | c. ■

Teorema 3.3.4 Sejam a, b ∈ Z não ambos nulos. Se d = (a, b), então


 
a b
, = 1.
d d

41
Capítulo 3. Divisão Euclidiana

Prova Como d = (a, b), existem x, y tais que d = ax + by. Como d | a


e d | b, temos
a b
1= x + y,
d d
do que segue-se  
a b
, = 1,
d d
como queríamos. ■

Exemplo 3.3.2 Vemos que 6 = (12, 18) resulta em 1 = (2, 3).

Teorema 3.3.5 Sejam a, b inteiros, não ambos nulo e n ∈ N. Então,

(na, nb) = n(a, b).

Prova Pelo teorema 3.2.2, temos que (na, nb) é o menor valor positivo
da forma anx + bny, logo

(na, nb) = n(ax + by).

Como (a, b) é o menor inteiro positivo da forma ax + by, vemos que

(na, nb) = n(a, b).

Portanto, o teorema é verdadeiro. ■

Exercícios

Exercício 3.3.1 Sejam a, b inteiros não ambos nulos e n ̸= 0. Mostre


que (na, nb) = |n|(a, b).

Exercício 3.3.2 Sejam a, b, c inteiros, com a, b não ambos nulos. Mostre


que, b | a e c | a se, e somente se bc
d | a, em que d = (b, c).

42
Capítulo 3. Divisão Euclidiana

Exercício 3.3.3 Seja n ̸= 0. Mostre que, se (a, b) = 1, então (an , bn ) = 1.

Exercício 3.3.4 Mostre que, se (a, b) = 1, então (ac, b) = (c, b).

Exercício 3.3.5 Sejam a, b, c inteiros, com a, b não ambos nulos. Mostre


que, a | bc se, somente se, a
d | c, em que d = (a, b).

3.4 O Mínimo Múltiplo Comum

O mínimo múltiplo comum raramente é usado, mas ainda assim


iremos fazer uma breve abordagem deste.

Definição 3.4.1 (Múltiplo comum) Sejam a, b, c inteiros. O número c é


tido ser um múltiplos comum de a, b se tivemos a | c e b | c.

O número 18 é múltiplo comum de 3 e 6, pois 3 | 18 e 6 | 18.


Pela definição acima fica claro que 0 é múltiplo comum de a e b.
Este é um caso trivial. Para ver que outros casos acontecem, percebe-se,
que −ab e ab são, ambos, múltiplos comuns de a e b, pois a, b | −ab
e a, b | ab, em que um deles é positivo. Propomos a seguinte definição.

Definição 3.4.2 (Mínimo múltiplo comum) Sejam a, b inteiros não nulos.


Chamamos de mínimo múltiplo comum de a e b, denotado por [a, b], o
inteiro positivo m tal que

i) a | m e b | m; e

ii) se a | c e b | c, então m | c.

Encurtaremos a palavra mínimo múltiplo comum pela abreviação


mmc.

43
Capítulo 3. Divisão Euclidiana

Proposição 3.4.1 Se m for o mmc de a, b, então é único.

Prova De fato, se m ′ for também mmc de a, b, então pelo item ii) da


definição, m ′ | m e m | m ′ . Como m, m ′ > 0, só podemos ter m = m ′ .

Exemplo 3.4.1 Para a = −12 e b = 30, temos que [−12, 30] = 60 é o


mmc de a, b.

Proposição 3.4.2 Sejam a, b inteiros, não ambos nulos. Se d = (a, b) e


m = [a, b], então dm = ab.

Prova Sem perda de generalidade, suponhamos a, b > 0. Coloquemos


ab
m= d . Como
b a
m=a =b ,
d d
temos que a | m e b | m; isto é, m é múltiplo comum de a e b.
Digamos agora que m ′ é outro múltiplo comum de a, b, isto é,
m ′ = aq = bq ′ . Segue-se que

a b
q = q′ .
d d
a b a
| q′ e b
| q. Deste

Por sorte de termos d, d = 1, isto implica em d d
modo m = a
db | bq = m ′ .
′ Concluímos assim, que m é o mmc entre
a, b e, portanto, dm = |ab|. ■

Exemplo 3.4.2 Como (−12, 18) = 6, temos que [−12, 18] = 12·18
6 = 36.
ab
Outra consequência interessante do fato que [a, b] = (a,b) é que, se
[a, b] | c, temos que a | c e b | c. De fato, temos igualdade [a, b]k = c,

44
Capítulo 3. Divisão Euclidiana

a b
para algum k ∈ Z. Como (a,b) e (a,b) são inteiros, vemos que
   
ab b a
k=a k =b k = c.
(a, b) (a, b) (a, b)

Logo, a | c e b | c.

Proposição 3.4.3 Sejam a, b, c inteiros não nulos. Se a | c e b | c, então


[a, b] = m | c.

Prova Suponhamos que m ∤ c. Pelo algoritmo da divisão existem q, r


tais que c = mq + r, com 0 ⩽ r < m. Uma vez que r = c − mq e a | c e
a | m, temos a | r. Analogamente, vemos que b | r. Consequentemente,
r é um múltiplo comum de a e b, o que é impossível, pois m é o menor
múltiplo comum de a e b. Só podemos acreditar que m | c. ■
A proposição acima nos permite demonstrar seguinte teorema.

Teorema 3.4.1 Sejam m1 , m2 , . . . , mr inteiros positivos. Então,

[m1 , m2 , . . . , mr ] = [[m1 , m2 , . . . , mr−1 ], mr ].

Prova Pondo m = [m1 , m2 , . . . , mr ] e m ′ = [[m1 , m2 , . . . , mr−1 ], mr ],


mostraremos que m | m ′ e m ′ | m.
Desde que m = [m1 , m2 , . . . , mr ] vemos que mi | m, pela proposição
3.4.3, fica claro que [m1 , m2 , . . . , mr−1 ] | m e mr | m. Assim, m | m ′ .
Agora, se [m1 , m2 , . . . , mr−1 ] | m ′ e mr | m ′ , cada mi | m ′ , fica
claro que, m | m ′ . ■

Exercícios

Exercício 3.4.1 Calcule [−36, −180].

45
Capítulo 3. Divisão Euclidiana

Exercício 3.4.2 Mostre que [a, b] = [−a, b] = [a, −b] = [−a, −b].

Exercício 3.4.3 Mostre que, se (a, b) = 1, então [a, b] = ab.

Exercício 3.4.4 Mostre que [na, nb] = |n|[a, b].

Exercício 3.4.5 Calcule [n + 1, n2 + 1], para todo n ⩾ 1.

Exercício 3.4.6 Mostre que [an , bn ] = [a, b]n , para todo n ⩾ 1.

46
4
Os Números Primos

Este capítulo será dedicado ao estudo dos números primos.

4.1 Fatoração Primária

Definição 4.1.1 (Número primo) Seja p um número natural maior do que


1. O número p é dito primo se seus únicos divisores são p e 1.

São consequências das definição acima os dois resultados que prova-


remos logo abaixo.

Proposição 4.1.1 Sejam p, q números primos. Se p | q, então p = q.

Prova De fato, se tivermos que p | q, pelo fato de q ser primo, temos


p = 1 ou p = q. Como p também é primo, isto é, p > 1, devemos ter
p = q. ■

Proposição 4.1.2 Seja p um número primo. Se a é um inteiro qualquer,


tal que p ∤ a, então (a, p) = 1.
Capítulo 4. Os Números Primos

Prova Suponhamos (a, p) = d. Logo, d | a e d | p; mas como p é


primo, devemos ter d = 1 ou d = p. No entanto, pelo fato de que
p ∤ a, devemos ter não podemos ter d = p, nos restando assim, d = 1;
consequentemente, (a, p) = 1. ■

Definição 4.1.2 (Número composto) Seja a um número natural maior do


que 1. O número a é dito composto se não for primo.

A respeito da definição acima, se a for composto, então existem b, c


inteiros, tais que a = bc, com 1 < b, c < a.

Exemplo 4.1.1 Os número 2, 3, 5 e 7 são primos, enquanto que 4, 6, 8 e 9


são compostos.

Teorema 4.1.1 Seja p um número primo, tal que p | ab. Então p | a ou


p | b.

Prova Se p | a, não há o que demonstrar; suponhamos assim que p | ab,


mas p ∤ a. Neste caso, por p ser primo e p ∤ a, em virtude da proposição
4.1.2, segue-se que (a, p) = 1. Sabendo agora que p | ab e (a, p) = 1,
em decorrência do Lema de Euclides (teorema 3.3.1), obtemos p | b. ■
Este teorema tem ainda uma versão mais geral ao produto de um
ou mais números. Vejamos o seguinte resultado.

Corolário 4.1.1 Seja p um número primo, tal que p | a1 a2 . . . an . Então


p | ai , para algum i = 1, 2, . . . , n.

Prova Provaremos por indução sobre n. Ora, se o número de fatores


for n = 1 não há o que discutir; e se for n = 2, pelo teorema acima,
temos o nosso resultado assegurado.

48
Capítulo 4. Os Números Primos

Suponhamos que, sempre que p divide um produto de menos do


que n fatores, este divide algum destes fatores. Deste modo, uma vez
que p | a1 a2 . . . an , pelo teorema 4.1.1, vemos que p | a1 a2 . . . an−1 ou
p | an .
Nossa hipótese assegura que p | a1 a2 . . . an−1 , porém, mesmo que
este não fosse o caso, ainda teríamos p | an . Em qualquer circunstância,
temos que p | a1 a2 . . . an . ■

Corolário 4.1.2 Sejam p, q1 , q2 , . . . , qn , todos números primos, tais que


p | q1 q2 . . . qn . Então p = qi , para algum i = 1, 2, . . . , n.

Prova Em virtude do corolário acima, sabemos que p | qi , para algum


i = 1, 2, . . . , n. Uma vez que p | qi , e tanto p quando qi são primos,
pela proposição 4.1.1, temos p = qi . Portanto, o resultado se confirma
verdadeiro. ■
Uma das coisas mais interessantes sobre número inteiros, em parti-
cular, dos números compostos, é que estes podem ser representado como
produto de fatores primos. Por exemplo, o número 24 pode ser escrito
como 24 = 2 · 2 · 2 · 3, ou abreviadamente, 24 = 23 · 3. Obviamente, se
não nos importarmos, podemos escrever também 24 = 3 · 2 · 2 · 2 = 3 · 22 ;
o que nos daria outra forma de escrita. No entanto, para nossos pro-
pósitos, seremos “organizados”, escreveremos os fatores em ordem, do
menor para o maior (o que é uma coisa bem inteligente a se fazer).
Lembremos que o número 1 não é primo e nem é composto; normal-
mente, este é chamado de unidade. Assim, este não aparece na escrita
em fatores de um número composto. Voltando ao caso do número
24, poderíamos, é claro, escrever 24 = 1 · 2 · 2 · 2 · 3, no entanto isto
não representa nenhum ganho, de modo que este não aparecerá nas
decomposições, que apresentamos agora no teorema abaixo.

49
Capítulo 4. Os Números Primos

Teorema 4.1.2 (Teorema Fundamental da Matemática) Sejam p1 , p2 , . . . , pk


números primos distintos. Então, todo número natural maior do que 1
pode ser escrito, de modo único, como um produto dos pi ’s, na forma

pα1 α2 αk
1 p2 . . . pk ,

em que k, αi > 0 e p1 < p2 < . . . < pk .

Prova Provaremos por indução sobre n. Além disso, organizaremos


nossa demonstração em duas partes: a escrita e a unicidade.

i) Escrita:

Se n for um número primo ou mesmo uma potência de primos não


temos o que demonstrar. Suponhamos assim que n seja composto e
que o teorema seja válido para todo número inteiro menor do que n.
Assim, podemos escrever n = ab, com 1 < a, b < n.
Como a e b satisfazem a nossa hipótese de indução, podemos escrever
a = pβ 1 β2 βk γ1 γ2 γk
1 p2 . . . pk e b = p1 p2 . . . pk . Agora, para cada i, somamos
as potências dos pi ’s que aparecem nas escritas de a e b, obtendo a
potência αi . Deste modo, podemos escrever

n = pα1 α2 αk
1 p2 . . . pk .

ii) Unicidade:

Suponhamos que podemos escrever

β1 β2
n = pα1 α2 αr βs
1 p2 . . . pr = q1 q2 . . . q s ,

em que os p’s e os q’s, em suas ordens, são todos primos distintos.


Mostraremos que r = s.

50
Capítulo 4. Os Números Primos

Note que p1 divide ambos os membros da igualdade acima; logo, pelo


β β
corolário 4.1.1, p1 divide qj j , para algum j. Como qj j = q1 q2 . . . qt ;
vemos, pelo corolário 4.1.2, que p1 = qj ; digamos p1 = j1 . Assim,
podemos escrever

β1 β2
pα1 α2 αr βs
1 p2 . . . p r = p1 q 2 . . . q s .

Se α1 > β1 , cancelamos pβ 1
1 de ambos os membros da igualdade acima,
obtendo

1
1 −β1 α2
p2 . . . p αr β2 β3 βs
r = q2 q3 . . . q s ,

que, por sua vez, é impossível porque agora p1 divide o lado esquerdo,
mas não o direito da igualdade acima. Pela mesma razão α1 < β1 não
pode acontecer, e assim, segue-se α1 = β1 . Agora cancelando p1 de
ambos os membros da igualdade, obtemos

β2
pα2 αr βs
2 . . . pr = q2 . . . q s .

Procedendo da mesma maneira, sempre acharemos um p com a


mesma potência de um q. Deste modo, não podemos terminar este
procedimento com nenhum primo em qualquer um dos membros da
igualdade; caso contrário, teríamos um produto de fatores primos igual
a 1, o que não pode ocorrer. Nos resta assim concluir que r = s.
Concluímos assim o nosso teorema. ■

Observação 4.1.1 A fatoração de um número n negativo é simplesmente


(−1) “vezes” a fatoração de (−n), pois n = (−1)(−n)

Exemplo 4.1.2 Mostremos que 2 é irracional.
√ √
Solução Suponhamos o contrário, que 2 é racional, isto é, a = 2 · b,
em que a, b são inteiros com (a, b) = 1. Deste modo, a2 = 2b2 ; logo,

51
Capítulo 4. Os Números Primos

b | a2 . Se tivermos b > 1, pelo Teorema Fundamental da Aritmética,


existe um primo p da fatoração b tal que p | b. Como p | b e b | a2 ,
segue-se que p | a2 , e pelo corolário 4.1.1, p | a; assim, p ⩽ (a, b) = 1,
o que é uma contradição, a menos que b = 1, mas neste caso a2 = 2,

que é impossível. Portanto, não podemos ter 2 racional. ■
Apresentaremos mais dois resultados que são consequências do
Teorema Fundamental da Aritmética.

Teorema 4.1.3 Sejam p1 , p2 , . . . , pk primos, com p1 < p2 < . . . < pn e


n = pα1 α2 αk
1 p2 . . . pk um número natural, maior do que 1, na sua forma
fatorada. Se d divide n, então

d = pβ 1 β2 βk
1 p2 . . . p k ,

em que 0 ⩽ βi ⩽ αi , para i = 1, 2, . . . , k.

Prova Desde que d divide n, existe d ′ tal que n = dd ′ , com d > 1 e


d > 1. Pelo Teorema Fundamental da Aritmética podemos escrever
d = qγ 1 γ2 γr ′ λ1 λ2 λs
1 q2 . . . qr e d = z1 z2 . . . zs . Assim a igualdade

pβ 1 β2 βk γ 1 γ2 γr λ1 λ2 λs
1 p 2 . . . p k = q 1 q 2 . . . q r z1 z2 . . . z s

apresenta duas fatorações primárias de n. Como, a fatoração primária


de n é única, devemos ter que os q1 , q2 , . . . , qr e os z1 , z2 , . . . , zs devem
ser iguais a alguns dos primos p1 , p2 , . . . , pk ; agora, agrupando aqueles
que são iguais, devemos ter

d = pβ 1 β2 βk
1 p2 . . . p k ,

com β ⩾ 0. ■

52
Capítulo 4. Os Números Primos

β1 βk
Teorema 4.1.4 Sejam n = pα1 αk
1 p2 α2 . . . pk e m = p1 p2 β2 . . . pk .
Mostre que, se γi = min{αi , βi }, tem-se

(n, m) = pγ 1 γ2 γk
1 p2 . . . p k .

Prova De fato, pelo teorema 4.1.3, pγ 1 γ2 γk


1 p2 . . . pk divide n e m. Se, por
sua vez, d ′ = pλ1 1 pλ2 2 . . . pλkk for outro divisor de n e m, é claro que λi =
min{αi , βi }, isto é, d ′ | n e d ′ | m. Portanto (n, m) = pγ 1 γ2 γk
1 p2 . . . pk .

Exercícios

Exercício 4.1.1 Mostre que, qualquer primo da forma 3m + 1 também é


da forma 6n + 1.

Exercício 4.1.2 Mostre que, se p é primo e p | an , então pn | an .

Exercício 4.1.3 Seja p um número primo, tal que p = (a, b). Determine
os possíveis valores de (a2 , b).

Exercício 4.1.4 Determine todos os pares de primos p, q tais que p−q = 3.

Exercício 4.1.5 Determine d = (138, 974).

Exercício 4.1.6 Determine todos os inteiros positivos a tais que a, a +


2 e a + 4 são primos.

Exercício 4.1.7 Mostre que todo número primo maior do que 3 é da


forma 6k − 1 ou 6k + 1, para k inteiro.

Exercício 4.1.8 Mostre que o único primo da forma a3 − 1 é o número 7.

Exercício 4.1.9 Mostre que se tivermos d = pβ 1 β2 βk


1 p2 . . . p k e n =
pα1 α2 αk
1 p2 . . . pk , com 0 ⩽ β ⩽ α, então d | n.

53
Capítulo 4. Os Números Primos

β1 βk
Exercício 4.1.10 Sejam n = pα1 αk
1 p2 α2 . . . pk e m = p1 p2 β2 . . . pk .
Mostre que, se γi = max{αi , βi }, tem-se

[n, m] = pγ 1 γ2 γk
1 p2 . . . p k .

4.2 Determinação dos Fatores Primos

Voltando um pouco ao Teorema Fundamental da Aritmética, vemos


que este nos diz que existe uma fatoração primária para números
maiores que do 1. Porém, um número n, não nos é dado nenhum
método de testar a divisibilidade deste por inteiros menores do que
ele. Apresentaremos agora um método que nos auxiliará neste árduo
trabalho.

Teorema 4.2.1 Seja n um número natural composto. Então, n tem um



divisor primo, tal que p ⩽ n.

Prova Como n é composto, existem a, b inteiros, tais que n = ab, com


1 < a, b < n. Suponhamos ter a ⩽ b. Assim,

a2 ⩽ ab = n,


isto é, a ⩽ n.
Por termos a > 1, sabemos, pelo Teorema Fundamental da Aritmé-
tica, que existe um primo p que configura na sua fatoração primária

de a, de modo que, p ⩽ a ⩽ n. Agora, veja que, como p | a e a | n,
segue-se que p | n. Portanto, existe um primo p divisor de n, tal que

p ⩽ n. ■

Exemplo 4.2.1 Encontremos os fatores primos que configura na fatoração


primária de n = 316.

54
Capítulo 4. Os Números Primos


Solução Temos 316 = 17,776 . . .. Dos primos menores do que ou
iguais a 17 somente 2 divide 316, isto é, 316 = 2 · 158. Para 158, temos

158 = 12,567 . . .. Dos primos menores do que ou iguais a 12, apenas
2 divide 158, isto é, 158 = 2 · 79. Como 79 é primo (faça o teste), vemos
que 316 = 2 · 2 · 79 = 22 · 79. ■
O resultado acima também nos apresenta um teste de primalidade,

pois se n não é divisível por nenhum primo p tal que p ⩽ n, este não
é composto, logo, é primo.

O Crivo de Eratóstenes

Vimos que para terminar todos os primos da fatoração de um número


n exige bastante tempo, por ser um processo trabalho. Assim, é de
grande ajuda termos uma tabela com números primos até uma certa
ordem; esta tão grande quando queiramos.
O matemático grego chamado Eratóstenes desenvolveu um algoritmo
que nos permite a construção de uma tal tabela de número primos,
dada pelos seguintes procedimentos. Considere o número n

i) determinemos todos os primos p ⩽ n;

ii) escrevemos em ordem crescente todos inteiros entre 2 e n;



iii) eliminamos todos os múltiplos de pi tais pi ⩽ n, exceto o
próprio pi ;

iv) os inteiros restantes são todos primos.

Exemplo 4.2.2 Vamos construir uma tabela de primos menores do que


ou iguais a n = 30.

55
Capítulo 4. Os Números Primos


Solução Temos que p ⩽ 30 = 5,477 . . ., assim, p = 2, 3, 5 e 7 são
primos. Agora seguindo os procedimentos acima e destacando na cor
vermelha os número eliminados no processo, obtemos

2 3 4 5 6 7 8 9 10
11 12 13 14 15 16 17 18 19 20
21 22 23 24 25 26 27 28 29 30

que é a tabela procurada, fornecendo os número primos menores do que


ou iguais a 30. ■
O resultado que apresentaremos agora foi demonstrado por Euclides
e apresenta uma das demonstrações mais bonitas, se não a mais bonita,
da história da matemática. Por favor, aprecie a beleza do argumento.

Teorema 4.2.2 Há infinitos números primos.

Prova Suponhamos, por absurdo, que exista uma quantidade finita


de números primos. Sejam p1 = 2, p2 = 3, . . . , pn os finitos números
primos ordenados em ordem crescente. Considere o inteiro positivo

P = p1 p2 . . . pn + 1.

Pelo Teorema Fundamental da Aritmética deve existir algum primo p


tal que p | P e p | p1 p2 . . . pn ; mas neste caso, p | P − p1 p2 . . . pn = 1,
o que é um absurdo. ■

Definição 4.2.1 (Primos gêmeos) Chamamos de primos gêmeos quaisquer


pares de primos consecutivos maiores do que 2.

Exemplo 4.2.3 São exemplos de primos gêmeos os pares 3 e 5, 5 e 7, 11


e 13, 17 e 19, etc..

56
Capítulo 4. Os Números Primos

O etc. acima não significa que estejamos afirmando aqui que há


infinitos pares deste tipo de primos, embora haja uma forte suspeita de
que isso seja verdade. O maior par deste tipo de primo encontrado até
hoje é 3756801695685 · 2666669 ± 1.
A respeito de haver ou não infinitos pares de primos gêmeos, pode-
mos construir uma sequência de números compostos determinada por
uma certa ordem de grandeza. Vejamos isso no nosso próximo teorema.

Teorema 4.2.3 Seja n um inteiro positivo qualquer. Então existem


sequências de n inteiros positivos, todos compostos.

Prova De fato, seja an = (n + 1)! + 1. Como os números 2, 3, . . . , (n + 1)


dividem (n + 1)!, vemos que

2 | (n + 1)! + 2 = an + 1
3 | (n + 1)! + 3 = an + 2
4 | (n + 1)! + 4 = an + 3
..
.
n | (n + 1)! + n = an + (n − 1)
(n + 1) | (n + 1)! + (n + 1) = an + n,

formam uma sequência crescente de n números compostos. ■

Exemplo 4.2.4 Para n = 4, temos obtemos a sequência 5! + 2, 5! + 3,


5! + 4 e 5! + 5, em que todos são compostos.

O teorema acima mostra que, dependendo da ordem de grandeza do


número n, pode ser bastante difícil encontrar um par de primos gêmeos.

57
Capítulo 4. Os Números Primos

Exercícios

Exercício 4.2.1 Chamamos de primos trigêmeos quaisquer trio de


primos consecutivos maiores do que 2. Mostre que 3, 5 e 7 é o único
trio deste tipo de números primos.

Exercício 4.2.2 Determinar a fatoração primária de 239, 253 e 5040



Exercício 4.2.3 Mostre que para qualquer número p o número p é
irracional.

Exercício 4.2.4 Determinar uma sequência de 6 números compostos.

Exercício 4.2.5 Mostre que o número 1987 é primo.

58
5
Funções

Este capítulo é dedicado ao estudo de algumas funções aritméticas


e da função maior inteiro.

5.1 Função Multiplicativa

Começaremos definindo o que é uma função aritmética, pois estas


desempenharão importantes nas próximas seções.

Definição 5.1.1 (Função aritmética) Uma função f é dita função aritmé-


tica se tem domínio em N e contradomínio em R.

Na verdade, a definição acima poder mais geral; podemos substituir


o conjunto R pelo conjunto C (dos números complexos). No entanto,
as funções que estudaremos nesta seção são aquelas cujo contradomínio
está em N.

Definição 5.1.2 (Função multiplicativa) Uma função aritmética f é dita


multiplicativa se, para m, n ∈ N, com (m, n) = 1, tem-se que f(m·n) =
f(m) · f(n).
Capítulo 5. Funções

Também podemos definir uma função completamente multiplica-


tiva se para todo m, n inteiros, tem-se f(m · n) = f(m) · f(n).

Exemplo 5.1.1 Mostremos que as funções constantes f(n) = 1 e identi-


dade g(n) = n, para todo n ∈ N, são multiplicativas.

Solução De fato, se f é constante, isto é, f(n) = 1, para todo n, temos


f(n · 1) = f(n) = f(n) · 1 = f(n) · f(1). Agora, se g é a função identidade,
isto é, g(n) = n, para todo n, temos g(n · 1) = g(n) = g(n) · 1 =
g(n) · g(1). ■

Proposição 5.1.1 Se f é uma função multiplicativa, então f(1) = 1.

Prova De fato, desde que f não seja a função identicamente nula, exite
n ∈ N tal que f(n) ̸= 0. Deste modo, f(n) = f(1 · n) = f(1) · f(n), o
que implica em f(1) = 1. ■

Teorema 5.1.1 Sejam n = pα1 α2 αk


1 p2 . . . pk , com p1 < p2 < . . . < pk pri-
mos, e f uma função aritmética, com f(1) = 1. Então é f é multiplicativa
se
f(pα1 α2 αk α1 α2 αk
1 p2 . . . pk ) = f(p1 )f(p2 ) . . . f(pk ).

Prova Provaremos por indução sobre k.


Se tivermos k = 1, é óbvio que a igualdade f(pα1 α1
1 ) = f(p1 ) vale,
pois f(pα α1 α1 α1 α1
1 ) = f(p1 · 1) = f(p1 ) · f(1) = f(p1 ) · 1 = f(p1 ).
1

Suponhamos que f(pα1 α2 αi α1 α2 αi


1 p2 . . . pi ) = f(p1 )f(p2 ) . . . f(pi ) seja
uma igualdade verdadeira, para todo inteiro positivo i menor do que k.

60
Capítulo 5. Funções

Deste modo,

α
f(pα1 α2 αk α1 α2 k−1 αk
1 p2 . . . pk ) = f(p1 p2 . . . pk−1 pk )
α
= f(pα1 α2 k−1 αk
1 p2 . . . pk−1 )f(pk )
α
= f(pα1 α2 k−1 αk
1 )f(p2 ) . . . f(pk−1 )f(pk ),

é uma igualdade verdadeira. Portanto, o teorema está assegurado. ■

Definição 5.1.3 (Função somativa) Seja f uma função aritmética. Chama-


mos de função somativa de f a função F tal que
X
F(n) = f(d),
d|n

em que d é um divisor de n.

Para deixar mais claro, a notação


X
f(d)
d|n

representa a soma dos valores f(d), de uma função aritmética f, sempre


que d percorrer o conjunto de todos os divisores positivos de n. Se, por
exemplo n = 10, temos
X
f(d) = f(1) + f(2) + f(5) + f(10).
d|n

Proposição 5.1.2 Sejam m, n números naturais, tais que (m, n) = 1. Se


o produto d1 d2 divide mn, com d1 | m e d2 | n, então (d1 , d1 ) =.

Prova Imagine que m = pα1 α2 αk α1 α2 αl


1 p2 . . . pk e n = q1 q2 . . . ql são
números naturais, com os pi ’s e os qj ’s primos distintos, isto é, (m, n) =

61
Capítulo 5. Funções

1. Como temos mn = pα1 α2 αk α1 α2 αl


1 p2 . . . pk q1 q2 . . . ql , vemos que se d
for um de seus divisores, também podemos escrever

d = pγ 1 γ2 γk λ1 λ2 λl
1 p2 . . . p k q 1 q 2 . . . q l ,

com 0 ⩽ γi ⩽ αi e 0 ⩽ λj ⩽ βj .
Pondo d = d1 d2 , tais que d1 = pγ 1 γ2 γk λ1 λ2 λl
1 p 2 . . . p k e d2 = q 1 q 2 . . . q l ,
vemos que d1 | m e d2 | n; além disso, como os pi ’s e os qj ’s são distintos,
só podemos ter (d1 , d2 ) = 1. ■

Teorema 5.1.2 Se f é uma função aritmética multiplicativa, então


X
F(n) = f(d)
d|n

também é multiplicativa.

Prova Sejam m, n inteiros positivos, com (m, n) = 1. Pondo


X
F(mn) = f(d)
d|mn

e sabendo que podemos escrever mn como um produto de inteiros d1 e


d2 , com d1 | m, d2 | n e (d1 , d2 ), temos
X
F(mn) = f(d1 d2 ).
d1 |m,d2 |n

62
Capítulo 5. Funções

Usando o fato de que f é multiplicativa, obtemos que


X
F(mn) = f(d1 d2 )
d1 |m,d2 |n
X
= f(d1 )f(d2 )
d1 |m,d2 |n
X X
= f(d1 ) f(d2 )
d1 |m d2 |n

= F(m)F(n),

como queríamos. Portanto F também é multiplicativa. ■

Exemplo 5.1.2 Vamos ilustrar o teorema acima, mostrando um caso


particular. Mostraremos que F(5 · 6) = F(5) · F(6).

Solução Temos
X
F(5 · 6) = f(d)
d|30

= f(1) + f(2) + f(3) + f(5) + f(6) + f(10) + f(15) + f(30)


= f(1 · 1) + f(2 · 1) + f(3 · 1) + f(5 · 1) + f(6 · 1) + f(5 · 2)
+ f(5 · 3) + f(6 · 5)
= f(1)f(1) + f(2)f(1) + f(3)f(1) + f(5)f(1) + f(6)f(1)
+ f(5)f(2) + f(5)f(3) + f(6)f(5)
= [f(1) + f(5)][f(1) + f(2) + f(3) + f(6)]
= F(5) · F(6),

como desejávamos. ■

63
Capítulo 5. Funções

Exercícios

Exercício 5.1.1 Mostre que, se f é uma função tal que f(pα1 α2 αk


1 p2 . . . pk ) =
f(pα1 α2 αk
1 )f(p2 ) . . . f(pk ), então f é multiplicativa.

Exercício 5.1.2 Seja g uma função aritmética tal que g(1) = 1 e sempre
que n > 1 tem-se g(n) = 0. Mostre que g é multiplicativa.

Exercício 5.1.3 Mostre que F(2 · 9) = F(2) · F(9).

5.2 Função Divisor e Soma de Divisores

Até agora não demos nenhum exemplo de função aritmética, tão


pouco de funções multiplicativas; faremos isso agora. Propomos a
seguinte definição.

Definição 5.2.1 (Função divisor) Seja n um número natural. Chamamos


de função divisor a função τ que conta o número τ(n) dos divisores
positivos de n.

A função divisor definida acima é um exemplo de função aritmética.


O símbolo τ é uma letra grega denominada tau.

Definição 5.2.2 (Número perfeito) Um número inteiro positivo n é dito


perfeito que n tem uma quantidade impar de divisores.

Exemplo 5.2.1 Se n = 16, temos τ(16) = 5, pois os divisores positivos


de 16 são 1, 2, 4, 8 e 16; isto é, 16 é um número perfeito.

Vimos acima que τ(16) = 5. Como 5 = 1 + 1 + 1 + 1 + 1, podemos


P
escrever τ(16) = d|16 f(d) = 1 + 1 + 1 + 1 + 1, em que f(1) = 1, f(2) =

64
Capítulo 5. Funções

1, f(4) = 1, f(8) = 1 e f(16) = 1. A função f neste caso é a função


constante, pois τ fará uma contagem de divisores. Em outras palavras,
X
τ(n) = 1
d|n

P
cujo significado de d|n 1 é a soma de tantos 1’s quanto for a quantidade
de divisores de n. Mostraremos abaixo que a função τ é multiplicativa,
com
X
τ(n) = f(d),
d|n

em que f(d) = 1, para todo divisor de n.

Teorema 5.2.1 Sejam m, n número naturais, com (m, n) = 1. Então, a


função τ é multiplicativa.

Prova Consideremos f(d) = 1 para todo divisor de mn. Já sabemos que


P
podemos escrever τ(n) = d|mn f(d). Deste modo, pela proposição
5.1.2, temos que existem d1 e d2 , tais que d1 | m, d2 | n com (d1 , d2 ) =
1; e pelo teorema teorema 5.1.2, vemos que
X X
τ(m · n) = f(d) = f(d1 · d2 )
d|mn d1 |m,d2 |n
X
= f(d1 ) · f(d2 )
d1 |m,d2 |n
X X
= f(d1 ) f(d2 )
d1 |m d2 |n

= τ(m) · τ(n),

como queríamos. ■

Exemplo 5.2.2 Se m = 5 e n = 6. Calculemos τ(30).

65
Capítulo 5. Funções

Solução Vejamos que τ(5) = 2 e τ(6) = 4; logo, τ(30) = τ(5 · 6) =


τ(5) · τ(6) = 2 · 4 = 8, pois como já sabemos, os divisores positivos de
30 são 1, 2, 3, 5, 610, 15 e 30. ■
É claro que calcular τ(n) usando o procedimento do teorema acima
pode ser um processo bastante trabalhoso; o ideal seria termos uma fór-
mula que nos permitisse fazer isso apenas olhando a fatoração primária
de n. Na verdade, isso é possível, como veremos logo mais.

Teorema 5.2.2 Seja n = pα1 α2 αk


1 p2 . . . pk um número natural, maior do
que 1, na sua forma fatorada. Então,

τ(n) = (α1 + 1)(α2 + 1) . . . (αk + 1)

Prova Primeiramente, vejamos que, se n = pα , em que p é primo,


os divisores positivos de n são 1, p, p2 , . . . , pβ . Neste caso, temos
τ(n) = α + 1, pois há α + 1 maneira de escolha para o expoente dos
divisores, já que 0 ⩽ β ⩽ α.
Como τ é multiplicativa vemos que

τ(pα1 α2 αk α1 α2 αk
1 p2 . . . pk ) = τ(p1 )τ(p2 ) . . . τ(pk ).

Uma vez que τ(pαi


i ) = αi + 1, temos

τ(n) = (α1 + 1)(α2 + 1) . . . (αk + 1),

como desejávamos. ■

Exemplo 5.2.3 Determinemos τ(756).

Solução Vejamos que 756 = 22 · 33 · 7. Deste modo, 756 tem τ(756) =


(2 + 1)(3 + 1)(1 + 1) = 3 · 4 · 2 = 24 divisores positivos. ■

66
Capítulo 5. Funções

Definição 5.2.3 (Função soma de divisores) Seja n um número natural.


Chamamos de função soma de divisores a função σ que soma os
divisores positivos de n, dada por σ(n).

A função soma de divisores também é uma função aritmética. O


símbolo σ é uma letra grega denominada sigma.

Exemplo 5.2.4 Se n = 18, temos σ(18) = 39, pois os divisores positivos


de 18 são 1, 2, 3, 6, 9 e 18.
P
Veja que, se escrevemos σ(18) = d|18 f(d) = 39, como 39 é soma
de 1, 2, 3, 6, 9 e 18, vemos que f é a função identidade, pois
X
f(d) = f(1) + f(2) + f(3) + f(6) + f(9) + f(18)
d|18

= 1 + 2 + 3 + 6 + 9 + 18.

Deste modo,
X
σ(n) = f(d),
d|n

em que f(d) = d.

Teorema 5.2.3 Sejam m, n número naturais, com (m, n) = 1. Então, a


função σ é uma função multiplicativa.

P
Prova Como σ(n) = d|n f(d) e f é a função identidade, que, por
sua vez é multiplicativa, pelo teorema 5.1.2, podemos concluir que
σ(m · m) = σ(m) · σ(n), sempre que (m, n) = 1. ■

Exemplo 5.2.5 Para m = 5 e n = 6, temos σ(5) = 6 e σ(6) = 12, assim,


σ(30) = σ(5) · σ(6) = 6 · 12 = 72.

67
Capítulo 5. Funções

Determinaremos uma fórmula para encontra soma de divisores n


no seguinte teorema.

Teorema 5.2.4 Seja n = pα1 α2 αk


1 p2 . . . pk um número natural, maior do
que 1, na sua forma fatorada. Então,

pα1 +1 − 1 pα2 +1 − 1 pαk +1 − 1


    
σ(n) = ... .
p1 − 1 p2 − 1 pk − 1

Prova Primeiramente, vejamos que, se n = pα , em que p é primo, os


divisores positivos de n são 1, p, p2 , . . . , pα ; pelo fato deste divisores
estarem em uma progressão aritmética, temos

pα+1 − 1
1 + p + p2 + . . . + pα = .
p−1

Assim, para cada i, temos

pα+1 −1
1 + pi + p2i + . . . + pα
i =
i
.
pi − 1

Agora, pelo fato de σ ser multiplicativa, isto é,

σ(pα1 α2 αk α1 α2 αk
1 p2 . . . pk ) = σ(p1 )σ(p2 ) . . . σ(pk ),

obtemos

pα1 +1 − 1 pα2 +1 − 1 pαk +1 − 1


    
σ(n) = ... ,
p1 − 1 p2 − 1 pk − 1

como desejado. ■

Exemplo 5.2.6 Para o número 360 = 23 · 32 · 5, temos que a soma


dos seus divisores
  2+1 positivos
 1+1  cuja soma destes é dada por σ(360) =
23+1 −1 3 −1 5 −1
2−1 3−1 5−1 = 15 · 13 · 6 = 1170.

68
Capítulo 5. Funções

Exercícios

Exercício 5.2.1 Determine τ(n) e σ(n), para

a) n = 1856; c) n = 3145;

b) n = 2514; d) n = 7654.

Exercício 5.2.2 Seja n um inteiro positivo. Mostre que, n é racional
se, e somente se, n é um quadrado perfeito; isto, é n = m2 para algum
inteiro m.

Exercício 5.2.3 Sejam a, b inteiros positivos, tais que (a, b) = 1. Mostre


que,

a) se ab é um quadrado, então, ambos, a e b também são;

b) se ab é uma n-ésima potência, então, ambos, a e b também são.

5.3 Função Maior Inteiro

Nosso principal interesse nesta seção é determinar a decomposição


primária de n!. Por exemplo, se n = 6, temos 6! = 24 · 32 · 5.

Definição 5.3.1 O número ep (m) indica o expoente do primo p na


decomposição primária de m.

No caso visto acima, temos e2 (6!) = 4, e3 (6!) = 2 e e5 (6!) = 1.


Além disso, como n! pode ser escrito sua fatoração primária da seguinte
forma
ep1 (n!) ep2 (n!) ep (n!)
n! = p1 p2 . . . pk k .

69
Capítulo 5. Funções

O algoritmo da divisão nos diz que dados inteiros a e b > 0 existem


únicos inteiros q e r tais que a = bq + r, com 0 ⩽ r < b. Não é difícil
de percebermos que q ⩽ a = bq + r e que bq + r ⩽ (q + 1)b = q + b,
isto é, q ⩽ a ⩽ (q + 1)b. Como b é um número inteiro positivo temos

a
q⩽ ⩽ q + 1.
b

Anotaremos por qb (a) para indicar que q é o quociente da divisão


de a por b.

Definição 5.3.2 (Função maior inteiro) Sejam a, b números inteiros, com


b ̸= 0. Chamamos de função maior inteiro, dada por qb (a) = a
 
b ,
a
a função que converte o número racional b no maior número inteiro
menor do que ou igual ele.

a a
Chamaremos b de parte inteiro de b. A propósito, poderíamos
ter posto uma definição com ⌊x⌋, para todo número real x, no entanto,
para os nossos fins, achamos que isso não seria necessário.

Exemplo 5.3.1 A parte inteiros dos números −5,78, 7,65, 32 e 10 são,


respectivamente, ⌊−5,78⌋ = −5, ⌊7,65⌋ = 7, 32 = 1 e ⌊10⌋ = 10.
 

Imagine, por exemplo, que queremos calcular a quantidade de múl-


tiplos 6 entre 1 e 100. Para isto, basta vermos que 100 = 6 · 16 + 4, do
qual podemos concluir que ente 1 e 100, os números

6 · 1, 6 · 2, . . . , 6 · 16

são os múltiplos procurados, numa quantidade de 16 números. De outro


modo, 100
 
6 = 16.
Sempre que tivermos dois inteiros a, b tais que 0 < b < a, a

70
Capítulo 5. Funções

quantidade de múltiplos não nulos de b menores do que ou iguais a a é


dado por a
 
b .
Mais interessante ainda, se tivermos inteiros a, b, c tais que 0 < c <
b < a, para calcularmos os múltiplos não nulos de c entre b e a, temos
a seguinte contagem:
a  b−1 
i) c − c , se b for múltiplo de c;
a b
ii) c − c , se b não for múltiplo de c.

Por favor, tente demonstrar que nossa afirmação feita acima é


verdadeira.

Exemplo 5.3.2 Determinemos quantos múltiplo de 9 há entre 279 e 870.

Solução Primeiramente, notemos que 279 = 9 · 31 é múltiplo de 9, assim


usaremos o item i) acima. Temos
   
870 279
− = 96 − 31 = 65,
9 9

isto é, há 65 múltiplos de 9 entre 279 e 870. ■


Mostraremos agora dois teoremas que nos permitirão determinar os
exponentes de um número primo que configura na decomposição de n!.

Teorema 5.3.1 Seja a, b e c inteiros positivos. Então,


$ %
a jak
b
= .
c bc

Prova Façamos
jak
a=b +r com 0 ⩽ r ⩽ b − 1.
b

71
Capítulo 5. Funções

Da mesma forma
$ %
jak a
=c b
+ r′ com 0 ⩽ r ′ ⩽ c − 1.
b c

Assim, vemos que


$ % !
a
b ′
a=b c +r +r
c
$ %
a
= bc b
+ br ′ + r.
c

Como de 0 ⩽ r ⩽ b − 1 e 0 ⩽ r ′ ⩽ c − 1, obtemos 0 ⩽ br ′ + r ⩽ bc − 1,
podemos concluir, portanto, que o nosso resultado é verdadeiro. ■

Teorema 5.3.2 Seja p um número primo que aparece na decomposição


primária de n!. Então,
∞ 
X 
n
ep (n!) = .
pi
i=1

Prova Pelo algoritmo da divisão podemos escrever


 
n
n=p +r com 0 ⩽ r < p.
p

O múltiplos de p entre 1 e n são


 
n
1p, 2p, . . . , p,
p

72
Capítulo 5. Funções

então,
 
n
n! = 1p · 2p · . . . · p · d1
p
 
n
! · p ⌊ p ⌋ · d1
n
=
p
j k
n
em que (p, d1 ) = 1. Os múltiplos de p entre 1 e p são

j k
 n 
 p 
1p, 2p, . . . ,   p.
p

Como, pelo teorema 5.3.2,


j k
 n   
= n ,
 p 

p p2

temos     j k
n n n
= 2
! · p p2 · d2 ,
p p
em que (p, d2 ) = 1. Assim
  j k
n ⌊ np ⌋+ pn2
n! = · p · d1 · d2 .
p2

Agora, procedendo por indução sobre n, temos

X
k 
n

ep (n!) = ,
pi
i=1

o que conclui o teorema. ■


j k
n
Para deixar claro, sempre que pi > n, temos pi
= 0, logo podemos

73
Capítulo 5. Funções

escrever
∞ 
X 
n
ep (n!) = .
pi
i=1

Exemplo 5.3.3 Para n = 10, temos 10! = 28 · 34 · 52 · 7. Veja que


   
10 10
e3 (10!) = + = 3 + 1 = 4.
3 9

Exemplo 5.3.4 Determinemos em quantos zeros termina o número 357!

Solução Claro que será igual ao número α tal que 10α | 357!. Como
10α = (2 · 5)α = 2α · 5α . Basta calcularmos o expoente de 5; no nosso
caso e5 (357!) = 87, isto é, 357! termina em 87 zeros. ■

Exercícios

Exercício 5.3.1 Determine quantos múltiplos de 7 há entre 100 e 1000.

Exercício 5.3.2 Determine a maior potência de 165 que divide 2000!.

Exercício 5.3.3 Determine a decomposição primária de 20!.

74
6
Equações Diofantinas

Neste capítulo estudaremos apenas equações diofantinas lineares


de duas variáveis.

6.1 Soluções Inteiras

Definição 6.1.1 Sejam a, b, c inteiros quaisquer. Uma equação diofantina


linear é qualquer combinação linear de a, b da forma

ax + by = c,

em que x, y ∈ Z.

O nosso principal interesse ao nos depararmos com uma equação é


determinar se esta tem ou não solução. No caso das equações diofantinas
estamos procurando inteiros x0 , y0 tais ax0 + by0 = c. Por exemplo,
o par 4, 1 satisfaz a equação 6x + 12y = 36; mas não somente este, os
pares −6, 6 e 10, −2 também.
Já adiantando, algumas equações diofantinas não tem solução, outras
apenas um número finito de soluções, e há também aquelas com infinitas
Capítulo 6. Equações Diofantinas

soluções. No nosso teorema abaixo daremos uma condição necessária e


suficiente para que uma equação diofantina seja solúvel em Z.

Teorema 6.1.1 Sejam a, b inteiros, não ambos nulos, com d = (a, b).
Então, a equação diofantina ax + by = c tem solução se, e somente, se
(a, b) | c.

Prova Consideremos x0 , y0 solução de ax+by = c, isto é, ax0 +by0 = c.


Como d = (a, b), vemos que a = dk e b = dk ′ , pois d | a e d | b.
Assim,

c = ax0 + by0 = (dk)x0 + (dk ′ )y0 = d(kx0 + k ′ y0 ),

ou seja, d | c.
Agora se d | c, temos c = dq, para algum q ∈ Z. Uma vez que
d = (a, b), sabemos que existem x ′ , y ′ tais que d = ax ′ + by ′ , logo,
dq = a(qx ′ ) + b(qy ′ ), isto é

c = ax0 + by0 ,

em que x0 = qx ′ , y0 = qy ′ é uma solução de ax + by = c. ■

Exemplo 6.1.1 A equação diofantina 6x + 13y = 8 tem solução, pois


(6, 13) = 1 | 8; já a equação 6x + 9y = 2 não tem, uma vez que
(6, 9) = 3 ∤ 2.

O teorema acima nos dá apenas condição sobre a existência ou


não de solução de ax + by = c, no entanto, este não nos diz como
encontrá-las. Nosso próximo resultado nos dirá como encontrar todas
as soluções de uma equação diofantina, caso exista, conhecendo-se uma
solução particular.

76
Capítulo 6. Equações Diofantinas

Teorema 6.1.2 Sejam a, b inteiros não nulos, com d = (a, b). Se x0 , y0


é uma solução particular de ax + by = c, então a solução gera é

x = x0 + b t
d
y = y − a t
0 d

com t ∈ Z.

Prova Uma vez que x0 , y0 é uma solução particular, sendo x, y outra


solução, temos
ax + by = ax0 + by0 = c.

Assim, a(x − x0 ) = b(y0 − y). Como d = (a, b), existem q, q ′ tais que
a = dq e b = dq ′ , com (q, q ′ ) = 1; logo,

q(x − x0 ) = q ′ (y0 − y).

Pelo Lema de Euclides, vemos que q ′ | x − x0 , isto é, x − x0 = q ′ t, para


t ∈ Z. Substituindo x − x0 = q ′ t em q(x − x0 ) = q ′ (y0 − y), obtemos
y0 − y = qt. Dado que q = a
d e q′ = b
d, temos

x = x0 + b t
d
y = y − a t
0 d

com t ∈ Z.
b
Obviamente que o par de número inteiros x = x0 + d t, y = y0 − a
dt
satisfaz a equação ax + by = c, pois
 
b  a 
a x0 + t + b y0 − t = ax0 + by0 = c.
d d

Concluímos assim o nosso resultado. ■

77
Capítulo 6. Equações Diofantinas

a b
Por favor, tenha cuidado com a escrita d e d, pois, neste caso, elas
não representam frações, mas sim números inteiros.

Exemplo 6.1.2 Determinemos as soluções da equação diofantina 4x+9y =


16.

Solução Como (4, 9) = 1 | 16, a equação tem solução. Usando o


Algoritmo de Euclides, temos 16 = 4(−32) + 9(16), em que x0 =
−32, y0 = 16 é uma solução particular. Assim, x = −32+9t, y = 16−4t,
para t ∈ Z, é a solução geral de 4x + 9y = 16. ■

Corolário 6.1.1 Sejam a, b inteiros não nulos, com (a, b) = 1. Se x0 , y0


é uma solução particular de ax + by = c, então a solução gera é

x = x0 + bt
y = y − at
0

com t ∈ Z.

Prova É uma repetição da demonstração do teorema 6.1.2. ■

Exercícios

Exercício 6.1.1 Resolva, se possível, as equações diofantinas abaixo.

a) 24x + 8y = 50; c) 2x + 14y = 17;

b) 17x + 6y = 4; d) 13x + 20y = 2.

Exercício 6.1.2 Encontre a única solução inteira positiva de 18x+5y = 48.

Exercício 6.1.3 Sejam a, b inteiros positivos, com (a, b) = 1. Mostre


que a equação diofantina ax − by = c tem infinitas solução positiva.

78
Capítulo 6. Equações Diofantinas

Exercício 6.1.4 Seja a, b inteiros, não ambos nulo, com (a, b) = 1.


Mostre que o par 
x = x0 + bt
y = y − at
0

é a solução geral de ax + by = c, com t ∈ Z e x0 , y0 uma solução


particular.

6.2 Soluções Não Negativas

Sempre que estivemos interessando nas soluções não negativas de


ax + by = c, podemos exigir x ⩾ 0, y ⩾ 0. No entanto, faremos uma
abordagem alternativa deste caso nesta seção.
Nosso primeiro resultado desta seção é, no mínimo, muito interes-
sante. Veremos como escrever qualquer número inteiro c conhecendo
apenas que (a, b) = 1, para a, b inteiros positivos. Faremos antes disso
uma construção particular da prova que daremos.
Tomemos os inteiros positivos a = 7, b = 4, em que (7, 4) = 1.
Sabemos que existem x, y tais que

7x + 4y = 1.

Para qualquer inteiro c, temos 7(cx)+4(cy) = c. Usando o algoritmo da


divisão podemos escrever cx = 4q + m, com 0 ⩽ m < 4. Substituindo
cx = 4q + m em 7(cx) + 4(cy) = c, obtemos 7m + 4n = c, com
0 ⩽ m < 4 e n = (4q + cy) ∈ Z. Em outras palavras, podemos escrever
qualquer inteiro c em função de inteiros a, b, tais que (a, b) = 1, com
algumas condições especiais.(Poderíamos, é claro, que ter escolhido cy).

79
Capítulo 6. Equações Diofantinas

Teorema 6.2.1 Sejam a, b inteiros positivos, com (a, b) = 1. Então, todo


inteiro c escrito, de modo único, como combinação linear de a, b na
forma
am + bn = c,

com 0 ⩽ m < b e n ∈ Z.

Prova Demonstraremos primeiro a existência da escrita, e, em seguida,


a sua unicidade.

i) Existência:

Uma vez que (a, b) = 1, sabemos que podemos escrever ax + by = 1,


para x, y ∈ Z. Deste modo, para qualquer inteiro c, temos a igualdade
a(cx) + b(cy) = c. Aplicando o algoritmo da divisão, em cx e b,
obtemos cx = bq + m, com 0 ⩽ m < b. Logo,

am + bn = c,

em que n = aq + cy é um inteiro qualquer e 0 ⩽ m < b.

i) Existência:

Se tivermos am + bn = am ′ + bn ′ , obtemos a(m − m ′ ) = b(n − n ′ ).


Além disso, |m − m ′ | < b, pois 0 ⩽ m, m ′ < b. Como (a, b) = 1, pelo
Lema de Euclides, vemos que b | m − m ′ ; logo, só podemos ter m = m ′ .
Assim, n = n ′ , o que nos permite concluir o nosso resultado. ■
Em consequência disso, o conjunto N0 dos inteiros não negativos
pode ser representado por

N0 = {am + bn; 0 ⩽ m < b e n ∈ Z}.

80
Capítulo 6. Equações Diofantinas

Uma equação diofantina ax + by = c, com a, b positivos e (a, b) = 1


certamente tem solução em N0 . Dividiremos N0 em dois subconjuntos
disjuntos, tal que em um deles há um número infinito de soluções, e no
outro uma quantidade limitada.

Definição 6.2.1 Sejam a, b inteiros positivos, com (a, b) = 1. Chamamos


de semi-gerado por a, b o conjunto de combinações lineares de a e b
dada por
S(a, b) = {ax + by; x, y ⩾ 0}.

No conjunto definido acima está a solução de algumas equações


diofantinas em que c ⩾ 0, mas por uma questão de má sorte nossa não
estão todas. O resultado abaixo nos dará a forma dos elementos de
S(a, b); ou de outra forma, nos permitirá dizer das equações diofantinas
tem solução em S(a, b).

Proposição 6.2.1 (Caracterização de S(a, b)) Sejam a, b inteiros positivos,


com (a, b) = 1. Então c ∈ S(a, b) se, e somente, se exitem únicos
inteiros não negativos m, n, tais que am + bn = c e 0 ⩽ m < b.

Prova Se c ∈ S(a, b), é óbvio que, pelo teorema 6.2.1, encontraremos


únicos inteiros não negativos m, n tais que am + bn = c e 0 ⩽ m < b.
Agora, se tivermos c = am + bn, com inteiros m, n não negativos e
0 ⩽ m < b, não há dúvidas que c ∈ S(a, b). ■

Exemplo 6.2.1 Seja S(a, b) o semi gerado de a, b. O número 18 =


7m + 4n, com 0 ⩽ m < 4 em que, m, n são não negativos, está em
S(a, b), pois 18 = 7(2) + 4(1). No entanto, o número 5 = 7m + 4n, pois
5 = 7(3) + 4(−4), com 0 ⩽ m = 3 < b = 4, mas n = −4 é negativo.

81
Capítulo 6. Equações Diofantinas

O exemplo acima mostra que algumas equações diofantinas com


c positivo não tem solução em N0 . Para ver isso, escrevamos 5 =
7(3) − 4(4), com 5 = 7(3) − 4(4), logo, 7m − 4n = 5 tem solução não
negativa. Podemos falar isso de outra maneira: os elementos de N0 ou
tem a forma c = am + bn, com 0 ⩽ m < b e n ⩾ 0, ou tem a forma
c = am − bn, com 0 < m < b e n ∈ N.

Definição 6.2.2 (Lacunas de S(a, b)) Sejam a, b inteiros positivos, com


(a, b) = 1. Chamamos de lacunas de S(a, b) o conjunto L(a, b) =
N0 \ S(a, b) formado por combinações lineares de a, b que não estão
em S(a, b).

Proposição 6.2.2 (Caracterização de L(a, b)) Sejam a, b inteiros positivos


com (a, b) = 1. Então,

L(a, b) = {am − bn; 0 < m < b e n > 0}.

Prova Suponhamos que c ∈ L(a, b), isto é, c ∈


/ S(a, b). Deste modo, pela
proposição 6.2.1, não existem inteiros m, n positivos com 0 < m < b.
No entanto, pelo teorema 6.2.1, existe um inteiro n ′ < 0, tal que
c = am + bn ′ , com 0 < m < b. Uma vez que n ′ é negativo, existe
n ∈ N tal que n = −n ′ , como c = am − b(−n ′ ), obtemos c = am − bn,
com 0 < m < b e n > 0. ■
Note que o conjunto L(a, b) é um conjunto finito. Este é limitado
inferiormente por 0 e tem um maior elemento que denotaremos por
max L(a, b). Neste caso, temos max L(a, b) = a(b−1)−b, que acontece
quando a diferença am−bn é a maior possível, ou seja, quando m = b−1
é máximo e n = 1 é mínimo. Como consequência disso, vemos que
todo c = am + bn ⩾ max L(a, b) + 1 esta em S(a, b), isto é, todo

82
Capítulo 6. Equações Diofantinas

c ⩾ (a − 1)(b − 1).

Exemplo 6.2.2 Determinemos para quais valore de c a equação diofantina


7m + 4n = c não tem solução em N0 .

Solução Os valores de c que procuramos estão em L(7, 4). Usando os


valores possíveis para m e n construímos a seguinte tabela:

m n 7m − 4n
1 1 7(1) − 4n = 3
2 1, 2, 3 7(2) − 4n = 10, 6, 2
3 1, 2, 3, 4, 5 7(3) − 4n = 17, 13, 9, 5, 1

Veja que L(7, 4) = {1, 2, 3, 5, 6, 9, 10, 13, 17}. Além disso, perceba que,
para todo c ⩾ (7 − 1)(4 − 1) = 18 os números da forma c = 7m + 4n
estão em S(a, b). ■
Perceba que N0 = S(a, b) ∪ L(a, b). (Essa união é disjunta.)

Teorema 6.2.2 Sejam a, b inteiros positivos, com (a, b) = 1. Uma


equação diofantina ax + by = c tem solução em N0 se, e somente se,
c∈
/ L(a, b).

Prova Supondo que ax + by = c tem solução em N0 é imediato que


c ∈ S(a, b), isto é, c ∈
/ L(a, b).
Se agora, c ∈
/ L(a, b), então, pela proposição 6.2.2, vemos que
c ∈ S(a, b), isto é, a equação ax + by = c tem solução em N0 . ■

Definição 6.2.3 (Solução minimal) Sejam a, b inteiros positivo, com


(a, b) = 1. O par m, n é dito ser a única solução minimal de ax+by =
c, com m < b se para qualquer outra solução x, y tivermos x ⩾ m.

83
Capítulo 6. Equações Diofantinas

Teorema 6.2.3 Sejam a, b inteiros positivos, com (a, b) = 1. Se o par


x0 , y0 é a solução minimal de ax + by = c, então a solução geral é

x = m + bt
y = n − at

com t ⩾ 0 e n − at ⩾ 0.

Prova Desde que x0 , y0 é a solução minimal, temos ax0 +by0 = c. Como


(a, b) = 1, sabemos que existem inteiros x ′ , y ′ tais que ax ′ − by ′ = 1;
logo, a(cx ′ ) + b(cy ′ ) = c. Usando o algoritmo da divisão podemos
escrever cx ′ = bq + m, com m < b. Assim, temos

am + b(aq − cy ′ ), ou
c=
am − b(cy ′ − aq).

No primeiro caso temos c ∈ S(a, b), se aq ⩾ cy, isto é, há solução em


N. No segundo caso temos c ∈ L(a, b), se cy ′ > aq, ou seja, não há
solução em N0 .
Agora, fazendo n = aq − cy ′ , como ax0 + by0 = am + bn = c, com
0 ⩽ m < b e n ⩾ 0, vemos que a solução geral é

x = m + bt
y = n − at

com t ⩾ 0 e n − at ⩾ 0. ■

Exemplo 6.2.3 Determinemos a solução minimal de 7x + 4y = 8

Solução Note que 11 ∈


/ L(7, 4), e como (7, 4) = 1 | 11, esta equação tem
solução não negativa.

84
Capítulo 6. Equações Diofantinas

Usando o algoritmo da divisão encontramos 11 = 7(−11) + 4(22).


Agora, dividindo −11 por 4, vemos que 11 = 7(4(−3) + 1) + 4(22) =
7(1) + 4(1), em que m = 1 e n = 1 é a solução minimal. A solução
geral é dada por x = 1 + 7t, y = 1 − 4t, com t ∈ N0 e 1 − 4t > 0. Além
disso, veja que, por termos 1 − 4t > 0, o parâmetro t só pode assumir o
valor 0, o que significa diz que, neste caso, a solução minimal é a única
solução. ■

Exercícios

Exercício 6.2.1 Suponha que num caixa eletrônico há apenas notas de 5


e 7 reais. Quais são as quantias que não se pode sacar com os valores
dessas notas? Qual é a quantidade mínima de notas de cada espécia
para se poder sacar 100 reais?

Exercício 6.2.2 Determine, em N0 , as soluções de

a) 8x + 13y = 23; c) 24x + 138y = 18;

b) 18x + 5y = 48; d) 16x + 7y = 36.

Exercício 6.2.3 Para quais valores de c a equação 14x + 18y = c não tem
solução em N0 .

Exercício 6.2.4 Um comerciante só aceita notas de 4 e 7. Quais são as


quantias que se pode pagar em seu estabelecimento comercial?

Exercício 6.2.5 Um cinema cobre 18 reais por adulto e 7,50 por criança.
Numa noite arrecadou-se 900 reais. Quantas crianças e adultos foram
ao cinema nessa noite?

85
7
Congruência dos Inteiros

Este capítulo será dedicado ao estudo da congruência de números


inteiros.

7.1 Números Congruentes

Definição 7.1.1 (Congruência) Sejam a, b, m inteiros quaisquer, com


m ̸= 0. Dizemos que os números a e b são congruentes segundo o
módulo m se m divide a diferença entre eles.

Com relação a definição acima, se m não dividir a diferença entre a


e b diremos que eles são incongruentes.
A noção de congruência vale para todos os inteiros, tanto os positivos
quanto para os negativos, mas não para as frações. Por exemplo, −6 e
10 são congruentes segundo o módulo 4. O número −3 é congruente a
18 segundo o módulo 7, mas não segundo o módulo 5.
Denotaremos a congruência dos números inteiros pelo símbolo “≡”,
e para dizermos que a e b são congruentes segundo o módulo m escreve-
remos a ≡ b (mod m). Além disso, leremos a escrita a ≡ b (mod m)
Capítulo 7. Congruência dos Inteiros

como: a é congruente a b módulo m. Caso m não divida a diferença


a − b escreveremos a ̸≡ b (mod m), para dizer que são incongruentes.
Numa congruência podemos trocar o módulo negativo por um
positivo. Por exemplo, tomemos a congruência 3 ≡ −5 (mod − 2),
isto é, −2 | 3 − (−5), já que −2(−4) = 3 − (−5); ou de outro modo,
2(4) = 3 − (−5), que significa 3 ≡ −5 (mod 2).
A partir de agora esteremos nos referindo sempre a módulos positivos.
Mais ainda, como 1 divide a qualquer diferença entre a e b iremos
descartar a congruência módulo 1, em outras palavras ao escrevermos
a ≡ b (mod m), tomaremos sempre m > 1.

Proposição 7.1.1 (Congruência é uma relação de equivalência) Sejam a, b, c, m


inteiros. Então,

i) a ≡ a (mod m);

ii) se a ≡ b (mod m), então b ≡ a (mod m);

iii) se a ≡ b (mod m) e b ≡ c (mod m), então a ≡ c (mod m).

Prova Seguiremos a ordem dos itens.

i) Se a ≡ a (mod m) é evidente que m | a − a;

ii) Se a ≡ b (mod m), temos m | a − b, então existe k ∈ Z tal que


a − b = mk, isto é, b − a = m(−k); logo m | b − a e assim b ≡ a
(mod m);

iii) Se a ≡ b (mod m) e b ≡ c (mod m), tem-se m | a−b e m | b−c;


consequentemente, existem k, k ′ ∈ Z tais que a − b = mk e
b − c = mk ′ . Deste modo, a − c = (a − b) + (b − c) = (k + k ′ )m.
Segue-se que m | a − c, e portanto, a ≡ c (mod m).

87
Capítulo 7. Congruência dos Inteiros

Concluímos assim a veracidade da nossa proposição. ■


Nosso próximo resultado nos dá uma condição necessária e suficiente
para verificar uma congruência.

Proposição 7.1.2 Sejam a e b inteiros. Então a ≡ b (mod m) se, e


somente se, é da forma b + km, para k ∈ Z.

Prova De fato, se a ≡ b (mod m) tem-se que m | a − b; assim, para


algum k ∈ Z, vemos que a − b = km, isto é, a = b + km.
Agora, se a é um inteiro da forma b + km, tem-se que a − b = km,
isto é, m | a − b, logo, a ≡ b (mod m). ■
A proposição acima nos permite escrever a = b + km, sempre que
a ≡ b (mod m). Se tomarmos b com 0 ⩽ b ⩽ m − 1 fica evidente a
relação entre uma congruência com o algoritmo da divisão. O nosso
próximo resultado caracteriza a relação de congruência em termos dos
restos da divisão por m, nos dando outra condição necessária e suficiente
para que dois inteiros a e b sejam congruentes.

Teorema 7.1.1 (Caracterização das Congruências) Dois inteiros quaisquer,


a e b, são congruentes módulo m se, somente se, deixa o mesmo resto
quando divididos por m.

Prova Supondo que a e b deixam o mesmo resto na divisão por m,


isto é, a = mq + r e b = mq ′ + r, com 0 ⩽ r ⩽ m − 1, temos
a − b = (mq + r) − (mq ′ + r) = m(q − q ′ ), isto é m | a − b, logo a ≡ b
(mod m).
Se agora tivemos a ≡ b (mod m), podemos escrever a = b + mk,
para k ∈ Z. Dividindo b por m, existem inteiros q e r tais que
b = mq + r, com 0 ⩽ r ⩽ m − 1. Assim, a = b + mk = (mq + r) + mk =
m(k + q) + r, isto é, a = mk ′ + r, com 0 ⩽ r ⩽ m − 1. ■

88
Capítulo 7. Congruência dos Inteiros

Em virtude do resultado acima, diremos que a e b são resíduos um


do outro sempre que a ≡ b (mod m).

Exemplo 7.1.1 Os números −19 e 29 são resíduos um do outro módulo


23, pois −19 = 3(−7) + 2 e 29 = 3(9) + 2 deixam o mesmo resto na
divisão por 3; isto é, −19 ≡ 29 (mod 23).

Uma vez que se a e b são congruentes módulo m, podemos limitar


b tal que este seja um dos inteiros 0, 1, 2 . . . , m − 1. Em outras palavras,
a é congruentes a algum dos m inteiros consecutivos 0, 1, 2, . . . , m − 1.
Provares abaixo um resultado mais geral.

Teorema 7.1.2 Seja a um número inteiro. Dados m inteiros consecutivos

b, b + 1, b + 2, . . . , b + (m − 1),

um e exatamente um deste é congruente a a módulo m.

Prova Sabemos, pelo algoritmo da divisão, que existem únicos inteiros


q, r tais que a − b = qm + r, com 0 ⩽ r ⩽ m − 1. Decorre disso que
b + r = a + (−q)m ≡ a (mod m) e b ⩽ b + r ⩽ b + (m − 1), logo,
b + r é o inteiro procurado. Deste modo, a é congruente a algum dos
m inteiros consecutivos dado.
Suponhamos agora que a − b = r + km, com 0 ⩽ r ⩽ m − 1. Como
a = (b + r) + km, temos b + r ≡ a (mod m). Se, por ventura, tivermos
b + ri ≡ a (mod m) e b + rj ≡ a (mod m), podemos concluir que
m | rj − ri . Mas, pelo fato de rj − ri ⩽ m − 1, devemos ter rj − ri = 0,
logo, ri = rj . Assim, concluímos que a só pode ser congruente a
exatamente um dos m inteiros consecutivos dado. ■
Chamaremos a qualquer conjunto de m inteiros consecutivos de sis-
tema de resíduo módulo m. Por exemplo o conjunto −4, −3, −2, −1, 0 é

89
Capítulo 7. Congruência dos Inteiros

um sistema de resíduo módulo 5; do mesmo modo, o conjunto 0, 1, 2, 3, 4.


Também é um sistema de resíduo módulo 5 o conjunto 3, 4, 5, 6, 7.
Note que devido ao teorema acima, sempre somos capazes de encon-
trar um conjunto de m inteiros, digamos a1 , a2 , . . . , am , tais que cada
um destes é congruente a exatamente um dos inteiros 0, 1, 2, . . . , m − 1.
Por exemplo se tomarmos o módulo 5, o conjunto −10, −19, 17, 13, 9
é tal que cada de seus elementos é congruente a exatamente um dos
resíduos 0, 1, 2, 3, 4, a saber:

−10 = 5(−2) + 0
−19 = 5(−4) + 1
17 = 5(3) + 2
13 = 5(2) + 3
9 = 5(1) + 4.

Propomos a seguinte definição.

Definição 7.1.2 (Sistema completo de resíduo) Chamamos de sistema


completo de resíduo módulo m a qualquer conjunto de m inteiros
incongruentes dois a dois módulo m.

É claro que o conjunto 0, 1, 2, . . . , m − 1 é um sistema completo de


resíduo.

Proposição 7.1.3 Seja a, b inteiros tais que a ≡ b (mod m). Então,


(a, m) = (b, m).

Prova Supondo a ≡ b (mod m), existe k ∈ Z tal que a = b+km. Deste


modo, pela proposição 3.2.1, não é difícil de perceber que (a, m) =
(b + km, m) = (b, m). ■

90
Capítulo 7. Congruência dos Inteiros

Já sabemos que num sistema completo de resíduo os seus elementos


são dois a dois incongruentes segundo o módulo m. Obviamente que,
se o módulo for um número composto, alguns destes elementos e o
módulo possuem algum fator comum; por sorte nossa, não são todos.
Por exemplo, o conjunto 0, 1, 2, 3, 4, 5 é um sistema completo de resíduo
módulo 6, e os resíduos 1 e 5 não tem fator em comum com 6, isto
é, (1, 6) = (5, 6) = 1. É claro que, no caso em que m é primo, todos
são primos com o módulo. Na definição que propomos abaixo estamos
interessados nos elementos de um sistema completo de resíduo que são
primos com o módulo.

Definição 7.1.3 (Sistema reduzido de resíduo) Um sistema reduzido de


resíduo módulo m é qualquer conjunto de inteiros primos com m e
incongruentes dois a dois.

Fica claro pela definição acima que o conjunto formado por 1 e 5 é


um sistema reduzido de resíduo módulo 6.

Exercícios

Exercício 7.1.1 Verifique se as congruências abaixo são satisfeitas.

a) 13 ≡ 1 (mod 2);

b) −2 ≡ −1 (mod 3);

c) 111 ≡ −9 (mod 40);

d) 22 ≡ 7 (mod 5).

Exercício 7.1.2 Encontre um sistema completo de resíduos negativos tais


que a congruência 7 ≡ r (mod 5).

91
Capítulo 7. Congruência dos Inteiros

Exercício 7.1.3 Determine m tal que

a) 27 ≡ 5 (mod m);

b) 13 ≡ 29 (mod m);

c) 1001 ≡ 0 (mod m);

d) 51 ≡ −9 (mod m).

Exercício 7.1.4 Encontre um sistema reduzido de resíduos negativos tais


que a congruência 5 ≡ r (mod 6).

Exercício 7.1.5 Encontre pelo menos dois sistemas reduzido de resíduo


módulo 8.

Exercício 7.1.6 Mostre que, se a ≡ b (mod m) e n | m, então a ≡ b


(mod n).

7.2 Propriedades da Congruência

Proposição 7.2.1 (Operações) Sejam a, b, c, d, m inteiros.

i) Se a ≡ b (mod m) e c ≡ d (mod m), então a + c ≡ b + d


(mod m);

ii) Se a ≡ b (mod m) e c ≡ d (mod m), então a·c ≡ b·d (mod m).

Prova Consideremos que a ≡ b (mod m) e c ≡ d (mod m) seja ver-


dade. Deste modo, m | a − b e m | c − d, logo,

i) m | (a − b) + (c − d) = (a + c) − (b + d), consequentemente
a + c ≡ b + d (mod m);

92
Capítulo 7. Congruência dos Inteiros

ii) m | (a−b)c e m | (c−d)b, assim m | (a−b)c+(c−d)b = ac−bd,


o que implica em a · c ≡ b · d (mod m).

Portanto, o resultado está concluído. ■


Uma das grandes vantagens de se trabalhar com congruências é
justamente o fato de que podemos usar somar e multiplicar.

Exemplo 7.2.1 Mostremos que, se a ≡ b (mod m), então an ≡ bn


(mod m).

Solução Usaremos indução sobre n. Obviamente que a1 ≡ b1 (mod m).


Supondo que an ≡ bn (mod m) seja verdade, vemos que an+1 =
an · a ≡ bn · b = bn+1 (mod m). Portanto, an ≡ bn (mod m). ■

Exemplo 7.2.2 Determinemos o resto da divisão de 520 por 4.

Solução Primeiramente vejamos que 5 ≡ −3 (mod 4), logo 52 ≡ 9


(mod 4). Como 9 ≡ 1 (mod 4), vemos que 52 ≡ 1 (mod 4). Assim,
520 ≡ 1 (mod 4), o que significa dizer 1 é o resto procurado. ■

Exemplo 7.2.3 Mostre que, se a + c ≡ b + c (mod m), então a ≡ b


(mod m).

Solução De fato, se a + c ≡ b + c (mod m), tem-se que m | (a + c) −


(b + c) = a − b, isto é, a ≡ b (mod m). ■
O exemplo acima mostra que a adição é cancelativa na relação de
congruência. Infelizmente, não podemos dizer o mesmo da multiplicação,
pois nem sempre é verdade que ac ≡ bc (mod m) implica em a ≡ b
(mod m). Por exemplo, 4 · 4 ≡ 4 · 1 (mod 8), mas 4 ̸≡ 1 (mod 8). No
entanto, guardada certas precauções podemos fazer o cancelamento na
multiplicação.

93
Capítulo 7. Congruência dos Inteiros

Proposição 7.2.2 Sejam a, b, c, m inteiros, com (c, m) = d. Se ac ≡ bc


m
(mod m), então a ≡ b (mod d ).

Prova Desde que ac ≡ bc (mod m) temos m | ac − bc = (a − b)c.


Deste modo, para algum k ∈ Z vemos que (a − b)c = mk, o que implica
em (a − b) dc = m
d k. Agora, pelo Lema de Euclides, como ( dc , m
d ) = 1,
segue que m
d | (a − b), logo, a ≡ b (mod m
d ). ■

Exemplo 7.2.4 Vemos que 8 · 5 ≡ 2 · 5 (mod 15); como (5, 15) = 5, temos
8 ≡ 2 (mod 3).

Não é difícil de perceber que, em consequência da proposição acima,


se ac ≡ bc (mod m) e (c, m) = 1, tem-se a ≡ b (mod m). Por
exemplo, 7 · 6 ≡ 7 · 1 (mod 5) e (7, 5) = 1, logo, 6 ≡ 1 (mod 5).

Teorema 7.2.1 Seja r1 , r2 , . . . , rm um sistema completo, ou reduzido, de


resíduo módulo m. Se (a, m) = 1, então ar1 + b, ar2 + b, . . . , arm + b
é um sistema completo, ou reduzido, de resíduo.

Prova Mostraremos que os elementos de ar1 + b, ar2 + b, . . . , arm + b


são dois a dois incongruentes módulo m.
Fazendo a soma da congruência ari + b ≡ arj + b (mod m) com
−b ≡ −b (mod m) obtemos ari ≡ arj (mod m). Agora, como (a, m) =
1, temos ri ≡ rj (mod m), o que implica i = j, pois os elementos de
r1 , r2 , . . . , rm formam um sistema completo, ou reduzido, de resíduo
módulo m. Logo, se i ̸= j vemos que os elementos ari + b e arj + b são
incongruentes. Portanto, ar1 + b, ar2 + b, . . . , arm + b é um sistema
completo, ou reduzido, de resíduo módulo m. ■

94
Capítulo 7. Congruência dos Inteiros

Proposição 7.2.3 Sejam a, b, m1 , m2 , . . . , mk inteiros tais que a ≡ b


(mod mi ). Então, a ≡ b (mod [m1 , m2 , . . . , mk ]).

Prova Desde que a ≡ b (mod mi ), temos mi | a − b, isto é, a − b é um


múltiplo de cada mi , logo [m1 , m2 , . . . , mk ] | a − b. Assim, segue-se
que a ≡ b (mod [m1 , m2 , . . . , mk ]) ■

Exercícios

Exercício 7.2.1 Mostre que 41 divide 220 − 1.

Exercício 7.2.2 Determine o resto da divisão

a) de 1! + 2! + . . . 99! + 100! por 12;

b) de 1! + 2! + . . . 99! + 1010 ! por 40;

c) de 21000 por 11;

d) de (116 + 171 7)21 por 20.

Exercício 7.2.3 Mostre que, para todo n natural

a) 92n ≡ 1 (mod 10);

b) 102n+1 ≡ −1 (mod 11).

Exercício 7.2.4 Mostre que, se a ≡ b (mod m) e c > 0, então a ≡ b


(mod cm).

Exercício 7.2.5 Mostre que a2 ≡ b2 (mod m) não implica em a ≡ b


(mod m).

Exercício 7.2.6 Mostre que, se a é ímpar, então a2 ≡ 1 (mod 8).

95
Capítulo 7. Congruência dos Inteiros

Exercício 7.2.7 Mostre que, para qualquer inteiro a, se a3 ≡ r (mod 7),


então r = 0, 1 ou 6.

Exercício 7.2.8 Mostre que, se ac ≡ bc (mod m) e (c, m) = 1, então


a ≡ b (mod m).

Exercício 7.2.9 Mostre que, se ac ≡ bc (mod m) e p ∤ c, em que p é


primo, então a ≡ b (mod m).

Exercício 7.2.10 Mostre que, se a ≡ b (mod m) e n | m, então a ≡ b


(mod n).

Exercício 7.2.11 Mostre que, se r1 , r2 , . . . , rm é um sistema completo, ou


reduzido, de resíduo módulo m e (a, m) = 1, então ar1 , ar2 , . . . , arm
também é um sistema completo, ou reduzido, de resíduo.

Exercício 7.2.12 Mostre que, se m1 , m2 , . . . , mk são primos entre si e


a ≡ b (mod mi ), então, a ≡ b (mod m1 · m2 · . . . · mk ).

7.3 Congruência Linear

Definição 7.3.1 Uma congruência linear é uma congruência da foram


ax ≡ b (mod m).

Nosso objetivo agora é encontrar os valores de x tais que ax ≡ b


(mod m) seja verdadeira. Começaremos estabelecendo as condições
para que uma congruência linear tenha solução. Antes disso, vejamos
que a congruência ax ≡ b (mod m) pode ser traduzida em termos
uma equação diofantina linear. Como m | ax − b, existe y tal que
ax − b = my, logo, ax + my = b.

96
Capítulo 7. Congruência dos Inteiros

Teorema 7.3.1 Uma congruência linear ax ≡ b (mod m) tem solução se,


e somente se, (a, m) | b.

Prova Suponhamos que x seja uma solução de ax ≡ b (mod m), isto


é, m | ax − b. Assim, existe y ∈ Z tal que ax − b = my, ou seja,
ax + my = b. Como a equação diofantina ax + my = b tem solução
x, y, vemos que (a, m) | b.
Agora se (a, m) | b, exite k tal que (a, m)k = b. Além disso, pela
Identidade de Bézout, existem x ′ , y ′ tais que ax ′ + my ′ = (a, m). Logo,
ax − b = my, em que x = kx ′ , y = −ky ′ , o que implica em m | ax − b.
Portanto, ax ≡ b (mod m) tem solução. ■

Exemplo 7.3.1 A congruência 3x ≡ 9 (mod 12) tem solução, pois


(3, 12) = 3 | 9. Em particular, x = −9 e 3 são soluções de 3x ≡ 9
(mod 12).

Veremos agora que, se (a, m) = d a congruência ax ≡ b (mod m)


tem exatamente d soluções.

Teorema 7.3.2 Sejam ax ≡ b (mod m) e (a, m) = d tal que d | b.


A congruência linear ax ≡ b (mod m) tem d soluções duas a duas
incongruentes da forma

m m m
x0 , x0 + , x0 + 2 , . . . , x0 + (d − 1) ,
d d d

em que x0 é uma solução particular.

Prova Mostraremos, primeiramente, que as soluções de ax ≡ b (mod m)


são da forma x = x0 + k m
d . De fato, como ax ≡ ax0 (mod m) e
m
(a, m) = d temos, pela proposição 7.2.2, que x ≡ x0 (mod d ). Assim,
podemos ver que x = x0 + k m
d , para algum k ∈ Z.

97
Capítulo 7. Congruência dos Inteiros

Dado que o algoritmo da divisão nos permite escrever k = dq + r,


para inteiros q, r, com 0 ⩽ r ⩽ d − 1, obtemos x = x0 + (dq + r) m
d =
x0 + dm + r m m
d ≡ x0 + r d (mod m), o que mostra que toda solução x
é congruente módulo m a um dos números

m m m
x0 , x0 + , x0 + 2 , . . . , x0 + (d − 1) .
d d d

Agora, se x0 + rj m m
d ≡ x0 + ri d (mod m), com 0 ⩽ ri < rj ⩽ d − 1,

d ≡ ri d (mod m), isto é, m | rj d − ri d . Uma vez que


temos rj m m m m

0 ⩽ ri < rj < d, vemos que 0 ⩽ ri m m m m


d < rj d < m, ou seja rj d − ri d <
m. Deste modo, rj m m
d − ri d = 0, o que implica em ri = rj . Logo, os
números x = x0 + r m
d são dois a dois incongruentes módulo m, o que
conclui o nosso teorema. ■

Exemplo 7.3.2 Determinemos as soluções de 6x ≡ 4 (mod 8).

Solução Por tentativa percebe-se que x0 = 2 é uma solução. Como


(6, 8) = 2, há duas soluções distintas, a saber: x = 2 e 2 + (2 − 1) 82 . ■
Como consequência do teorema acima, vemos que a congruência
ax ≡ b (mod m), com (a, m) = 1, tem uma única solução.
Consideremos a congruência linear ax ≡ 1 (mod m). Sabemos que
esta congruência tem solução se, e somente se, (a, m) = d | 1, o que
neste caso força d = 1. Propomos a seguinte definição.

Definição 7.3.2 (Inverso módulo m) Chamamos de inverso de a módulo


m, denotado por a−1 , a qualquer solução da congruência linear ax ≡ 1
(mod m).

Exemplo 7.3.3 O inteiro 9 é um inverso de 7 módulo 31, pois 9 é solução


de 7x ≡ 1 (mod 31). Do mesmo modo, 7 é um inverso de 9 módulo 31.

98
Capítulo 7. Congruência dos Inteiros

Conhecer o inverso de a módulo m pode encurtar nosso caminha na


busca de solução da congruência ax ≡ b (mod m). Desde que aa−1 ≡ 1
(mod m) e ax ≡ b (mod m), podemos fazer x ≡ (aa−1 )x ≡ a−1 b
(mod m), isto é, x ≡ a−1 b (mod m).

Exemplo 7.3.4 Determinemos a solução de 11x ≡ 6 (mod 8).

Solução Primeiramente, vejamos que a−1 = 3 é inverso de 11 módulo


8, pois 11(3) ≡ 1 (mod 8). Assim, x ≡ 3 · 6 = 18 ≡ 6 (mod 8). ■

Exercícios

Exercício 7.3.1 Determine as soluções de

a) 9x ≡ 5 (mod 25);

b) 6x ≡ 3 (mod 9);

c) 6x ≡ 21 (mod 18);

d) 3x ≡ 5 (mod 7);

e) 34x ≡ 60 (mod 98);

f) 25x ≡ 15 (mod 29);

g) 12x ≡ −36 (mod 28).

Exercício 7.3.2 Mostre que, se (a, m) = 1 a congruência ax ≡ b (mod m)


tem uma única solução.

Exercício 7.3.3 Mostre que, se r é um elemento de um sistema reduzido


de resíduo, a congruência rx ≡ b (mod m) tem um única solução.

Exercício 7.3.4 Mostre que, se a−1 e b−1 forem, respectivamente, inversos


módulo m de a e b, então a−1 b−1 é inverso de módulo m de ab.

99
Capítulo 7. Congruência dos Inteiros

7.4 O Teorema Chinês dos Restos

Nosso objetivo agora é encontrar um método que nos permita de-


terminar a solução de um sistema de congruência linear. Por exemplo,
consideremos as congruências

x≡2 (mod 3)
x≡3 (mod 5)
x≡2 (mod 7)

obtido do problema de em que se pede para determinar o número


que deixa resto 2, 3 e 2 quando dividido, respectivamente, por 3, 5 e 7,
proposto pelo matemática chinês Sun-Tsu. (Por tentativa vê-se que 23
satisfaz as três congruências).

Teorema 7.4.1 (Teorema chinês dos restos) Sejam m1 , m2 , . . . , mr inteiros


primos entre si. Então, o sistema de congruência linear

x ≡ b1 (mod m1 )
x ≡ b2 (mod m2 )
..
.
x ≡ br (mod mr )

tem uma única solução módulo m = m1 m2 . . . mr , dada por

x = b1 M1 y1 + b2 M2 y2 + . . . + br Mr yr ,

em que Mi = m/mi e yi é solução de Mi y ≡ 1 (mod mi ).

Prova Iremos construir uma solução que satisfaça simultaneamente

100
Capítulo 7. Congruência dos Inteiros

cada uma das r equações do sistema. Primeiramente, consideremos


o produto m = m1 m2 . . . mr , do qual decorre que Mi = m/mi =
m1 m2 . . . mi−1 mi+1 . . . mr . Como os mi ’s são primos entre si, te-
mos (mi , Mi ) = 1, o que nos permite determinar um inverso yi da
congruência Mi y ≡ 1 (mod mi ) e escrever a soma

x = b1 M1 y1 + b2 M2 y2 + . . . + br Mr yr .

Agora mostraremos que x satisfaz cada uma das r congruências.


Note que, para i ̸= j, temos Mj ≡ 0 (mod mi ), pois mi | Mj . Resulta
disso que

x = b1 M1 y1 + b2 M2 y2 + . . . + br Mr yr ≡ bi Mi yi (mod mi ).

Como yi satisfaz a congruência Mi y ≡ 1 (mod mi ) podemos concluir


que x ≡ bi Mi yi ≡ bi (mod mi ), isto é, x é solução de cada uma das r
congruências.
Por fim, se x ′ for qualquer outra solução simultânea das r congruên-
cias temos x ≡ x ′ ≡ bi (mod mi ), assim, mi | x − x ′ . Como os mi ’
são primos entre si, vemos que [m1 m2 . . . mr ] = m1 m2 . . . mr , e deste
modo, pela proposição 7.2.3, segue-se que x ≡ x ′ (mod m). ■

Exemplo 7.4.1 Mostremos como resolver o problema de Sun-Tsu, dada


pelo sistema

x≡2 (mod 3)
x≡3 (mod 5)
x≡2 (mod 7).

Solução Primeiramente, formemos o produto m = 3 · 5 · 7 = 105. Temos

101
Capítulo 7. Congruência dos Inteiros

M1 = 5 · 7 = 35, M2 = 3 · 7 = 21 e M3 = 3 · 5 = 15. Resolvendo


as congruências 35y1 ≡ 1 (mod 3), 21y2 ≡ 1 (mod 5) e 15y3 ≡ 1
(mod 7), obtemos y1 = 2, y2 = 1 e y3 = 1. Assim,

x = 2 · 35 · 2 + 3 · 21 · 1 + 3 · 15 · 1 = 233 ≡ 23 (mod 105),

o que nos dá 23 ≡ 2 (mod 3), 23 ≡ 3 (mod 5) e 23 ≡ 2 (mod 7). ■


Visto que o exemplo acima pode ser resolvido pelo Teorema Chinês
dos Restos, podemos, certamente, nos perguntar: e se os módulos não
forem primos entre si, como garantir que um sistema de congruências
tem solução?
Por enquanto daremos uma resposta para o seguinte sistema

x ≡ b1 (mod m1 )
x ≡ b2 (mod m2 ).

Mostraremos que este sistema tem única solução módulo m = [m1 , m2 ]


se, e somente se, b1 ≡ b2 (mod (m1 , m2 )).
Desde que x ≡ b1 (mod m1 ) podemos escrever x = b1 + m1 k.
Agora, substituindo x na segunda congruência, obtemos a congruência
linear m1 k ≡ b2 − b1 (mod m2 ) que, por sua vez, tem solução se, e
somente, se (m1 , m2 ) | b2 − b1 , isto é b1 ≡ b2 (mod (m1 , m2 )).
Se a é solução do sistema, qualquer outra solução a ′ é tal que
a ≡ a ′ ≡ b1 (mod m1 ) e a ≡ a ′ ≡ b2 (mod m2 ) o que implica em
a ≡ a ′ (mod [m1 , m2 ]).

102
Capítulo 7. Congruência dos Inteiros

Exemplo 7.4.2 Determinemos a solução do sistema

x≡4 (mod 8)
x≡6 (mod 6).

Solução Claro que (6, 8) = 2 | 6 − 2, logo o sistema tem solução.


Escrevendo x = 8k + 4 e substituindo em x ≡ 6 (mod 6), obtemos
8k ≡ 2 (mod 6), ou 2(4k) ≡ 2 · 1 (mod 6); daí, 4k ≡ 1 (mod 3). Uma
vez que 4k ≡ k (mod 3), podemos substituir 4k ≡ 1 (mod 3) por k ≡ 1
(mod 3). Agora, fazendo k = 3k ′ + 1, temos

x = 24k ′ + 12,

assim, x ≡ 12 (mod [6, 8]), pois [6, 8] = 24. ■


Para provarmos o caso geral do Teorema Chinês dos Restos, preci-
saremos do seguinte resultado.

Lema 7.4.1 Sejam m1 , m2 , . . . , mr , M inteiros. Então,

[(m1 , m), (m2 , m), . . . , (mr , m)] = ([m1 , m2 , . . . , mr ], m).

Prova Suponhamos que p um número primo divisor e α o maior


expoente de p tal que pα | m = [(m1 , M), (m2 , M), . . . , (mr , M)].
Neste caso, vemos que pα divide ao menos um dos (mi , M), diga-
mos (m1 , M). Logo, pα é um divisor comum de m1 e M. Desde que
pα | m1 , vemos que pα divide qualquer múltiplo de m1 ; em particular,
pα divide [m1 , m2 , . . . , mr ]. Segue-se que pα é um fator comum de
[m1 , m2 , . . . , mr ] e M, isto é, pα divide ([m1 , m2 , . . . , mr ], M).
Supondo agora que pα divide ([m1 , m2 , . . . , mr ], M), significa que
pα divide [m1 , m2 , . . . , mr ] e M. Assim, pα divide ao menos um dos

103
Capítulo 7. Congruência dos Inteiros

mi , digamos m1 . Sendo pα um divisor comum de m1 e M este divide


(m1 , M); Logo, pα divide qualquer múltiplo de (m1 , M), em particular,
[(m1 , M), (m2 , M), . . . , (mr , M)]. ■

Teorema 7.4.2 (Teorema Chinês dos Restos Generalizado) O sistema de


congruência linear

x ≡ b1 (mod m1 )
x ≡ b2 (mod m2 )
..
.
x ≡ br (mod mr )

tem uma única solução módulo m = [m1 m2 . . . mr ] se, e somente se,


bi ≡ bj (mod (mi , mj )).

Prova Provaremos por indução sobre r. Quando r = 2 já sabemos que


o nosso resultado está assegurado. Suponhamos que que o resultado
seja verdadeiro para todo inteiro menor do que r, isto é, que o sistema
tem única solução b módulo [m1 , m2 , . . . , mr−1 ]. Basta mostrarmos
agora que o sistema

x≡b (mod [m1 , m2 , . . . , mr−1 ])


x ≡ br (mod mr )

possui única solução módulo [m1 , m2 , . . . , mr ]. No entanto, sabemos


que isto acontece quando r = 2, logo,

b ≡ br (mod ([m1 , m2 , . . . , mr−1 ], mr )).

104
Capítulo 7. Congruência dos Inteiros

Como b ≡ bi (mod mi ), temos b ≡ bi (mod (mi , mr )). Por hipó-


tese de indução sabemos que br ≡ bi (mod (mi , mr )), logo, b ≡ br
(mod (mi , mr )). Assim, pelo lema 7.4.1 e teorema 3.4.1, temos

b ≡ br (mod [(m1 , mr ), (m2 , mr ), . . . , (mr−1 , mr )])


≡ br (mod ([m1 , m2 , . . . , mr−1 ], mr ))
≡ br (mod [m1 , m2 , . . . , mr ]).

Portanto, o sistema de r congruências lineares tem única solução módulo


m = [m1 , m2 , . . . , mr ]. ■

Exemplo 7.4.3 Determinemos x tal que

x≡5 (mod 6)
x≡3 (mod 10)
x≡8 (mod 15).

Solução Note que 5 ≡ 3 (mod (6, 10)), 5 ≡ 8 (mod (6, 15)) e 3 ≡ 8


(mod (10, 15)). Comecemo reescrevendox ≡ 8 (mod 15) como x =
15k + 8. Substituindo x nas duas primeiras congruências obtemos

15k ≡ −5 (mod 6)
15k ≡ −3 (mod 10).

Estas duas congruências equivalem a k ≡ −1 (mod 2), que por sua


pode ser reescrita como k = 2k ′ − 1. Assim, temos x = 30k ′ − 7; logo,
x ≡ −7 (mod 30). Uma vez que 23 ≡ −7 (mod 30), vemos que x ≡ 23
(mod 30). Interpretando 5, 3 e 8 como os respectivos restos das divisões
por 6, 10 e 15, vemos que faz sentido x = 23. (x = −7 também.) ■

105
Capítulo 7. Congruência dos Inteiros

Exercícios

Exercício 7.4.1 Resolva os sistemas abaixo.

a) x ≡ 1 (mod 3), x ≡ 2 (mod 5), x ≡ 2 (mod 7);

b) x ≡ 5 (mod 11), x ≡ 14 (mod 29), x ≡ 15 (mod 31);

c) x ≡ 5 (mod 14), x ≡ 15 (mod 20), x ≡ 10 (mod 30);

d) x ≡ 1 (mod 9), x ≡ 7 (mod 15), x ≡ 2 (mod 25).

Exercício 7.4.2 Determine o menos número natural que deixa resto 1, 3 e


quando divido, respectivamente, por 5, 7 e 9.

Exercício 7.4.3 Resolva a congruência 17x ≡ 3 (mod 2·3·5·7) resolvendo


o sistema

17x ≡ 3 (mod 2)
17x ≡ 3 (mod 3)
17x ≡ 3 (mod 5)
17x ≡ 3 (mod 7).

Exercício 7.4.4 Quais números inteiros deixam resto 1 quando divididos


por 2 e 3?

Exercício 7.4.5 Determine os inteiros que deixam resto 4 e 3 quando


divididos, respectivamente, por 11 e 17.

Exercício 7.4.6 Determine os inteiros que deixam resto 4 e 13 quando


divididos, respectivamente, por 6 e 15.

Exercício 7.4.7 Determine x tal que 13x ≡ 4 (mod 42).

106
Capítulo 7. Congruência dos Inteiros

7.5 Classes de Resíduos

Com o auxílio das congruências podemos particionar o conjunto dos


número inteiros em uma quantidade finita de subconjuntos disjuntos.
Por exemplo, consideremos o módulo 2 e a congruência a ≡ r (mod 2).
Uma vez que a = 2q + r e 0 ⩽ r < 2, temos apenas duas escolhas para r.
Em outras palavras, para um número a ser congruente a 2 este precisa
deixar resto 0 ou 1 quando dividido por 2. Os que deixam resto 0 são
os números pares, e os que deixam resto 1 são os ímpares. Propomos a
seguinte definição.

Definição 7.5.1 (Classe residual) Chamamos de classe residual módulo


m o conjunto de todos os inteiros que deixam resto r quando divididos
por m.

Perceba que pela definição acima que os possíveis valores que r pode
assumir são 0, 1, 2, . . . , m − 1; isto é, há m classes residuais módulo m.
Anotaremos por r = {a; a ≡ r (mod m)} a classe residual r, ou, em
outras palavras, o conjunto de todos os inteiros a que deixam resto r
quando divididos por m. Deste modo, as m classes residuais módulo
m são

0 = {a; a ≡ 0 (mod m)}


1 = {a; a ≡ 1 (mod m)}
2 = {a; a ≡ 2 (mod m)}
..
.
m − 1 = {a; a ≡ m − 1 (mod m)}.

Adotaremos o símbolo Zm para indicar o conjunto de todas as

107
Capítulo 7. Congruência dos Inteiros

classes de resíduos módulo m, isto é, Zm = {0, 1, 2, . . . , m − 1}. Por


exemplo, para m = 3, temos Z3 = {0, 1, 2}, em que

0 = {a; a ≡ 0 (mod 3)}


1 = {a; a ≡ 1 (mod 3)}
2 = {a; a ≡ 2 (mod 3)};

ou ainda,

0 = {a; a = 3q}
1 = {a; a = 3q + 1}
2 = {a; a = 3q + 2}.

Proposição 7.5.1 Sejam a, b números inteiros. Então,

i) a = b se, e somente, se a ≡ b (mod m);

ii) a = b se a ∩ b ̸= ∅;
S
iii) a∈N a = Z.

Prova É uma consequência da proposição 7.1.1. ■


Uma vez que r é um conjunto infinito, chamaremos de representante
de r a todo x ∈ Zm tal que x = r. Por exemplo, 2 ∈ Z3 e 8 é um
representante de 2, pois ambos deixam o mesmo resto na divisão por 3.
Devido ao algoritmo da divisão, não é difícil de se perceber que para
todo inteiro a existe um único inteiro r tal que a = r, pois a = mq + r,
com 0 ⩽ r < m. Como a − r = mq, temos a ≡ r (mod m), isto é,
a = r.
Vamos definir duas operações em Zm , as quais chamaremos de

108
Capítulo 7. Congruência dos Inteiros

adição e multiplicação. Dadas da seguinte maneira. Para todo a, b ∈


Zm , temos

i) a + b = a + b;

ii) a · b = a · b.

Exemplo 7.5.1 Para Z4 = {0, 1, 2, 3} determines as somas e as multipli-


cações de seus elementos.

Solução Faremos duas tabelas com todas as somas e multiplicações


possíveis de seus elementos. Temos

+ 0 1 2 3 · 0 1 2 3
0 0 1 2 3 0 0 0 0 0
1 1 2 3 0 1 0 1 2 3
2 2 3 0 1 2 0 2 0 2
3 3 0 1 2 3 0 3 2 1

em que a + b = a + b e a · b = a · b. ■

Exercícios

Exercício 7.5.1 Construa a tabela de adição e multiplicação de Z6 e Z7 .


(Em qual destes conjuntos todos os elementos não nulos são inversíveis?)

Exercício 7.5.2 Mostre que a adição em Zm é associativa, comutativa,


existe elemento neutro e inverso aditivo.

Exercício 7.5.3 Mostre que a multiplicação em Zm é associativa, comu-


tativa e existe elemento neutro.

Exercício 7.5.4 Mostre que um elemento a ∈ Zm é inversível se, e


somente se, (a, m) = 1.

109
Capítulo 7. Congruência dos Inteiros

Exercício 7.5.5 Mostre que, se m é primo então todo elemento em Zm


tem inverso multiplicativo.

Exercício 7.5.6 Mostre que Zm é um corpo se, e somente, se, m é primo.

Exercício 7.5.7 Mostre que, se ax ≡ 1 (mod m), então x é um inverso


de a.

110
8
Critérios de Divisibilidade

Neste capítulo veremos como representar um número inteiro na


formal polinomial e alguns critérios de divisibilidade.

8.1 Representação dos Inteiros

Teorema 8.1.1 Todo inteiro positivo a pode ser representado de modo


único na forma

a = an bn + an−1 bn−1 + . . . + a2 b2 + a1 b + a0 ,

em que b ⩾ 2, an ̸= 0, n ⩾ 0 e 0 ⩽ a0 , a1 , . . . , an < b.

Prova Provaremos a representação por indução sobre a. Quando a < b,


tomamos n = 0 e a0 = a.
Suponhamos a ⩾ b e que nosso resultado seja verdadeiro para
1, 2, . . . , a − 1. Pelo algoritmo da divisão podemos escrevemos a =
a−a0 a
bq+a0 , com 0 ⩽ a0 < b. Note que 0 < q < a, pois q = b ⩽ b < a,
Capítulo 8. Critérios de Divisibilidade

assim, por hipótese de indução, podemos escrever

q = an bn−1 + an−1 bn−2 + . . . + a2 b + a1 ,

e consequentemente

a = an bn + an−1 bn−1 + . . . + a2 b2 + a1 b + a0 .

Agora, suponhamos ter

a = an bn + an−1 bn−1 + . . . + a2 b2 + a1 b + a0
= cn bn + cn−1 bn−1 + . . . + c2 b2 + c1 b + c0 ,

com representações distintas. Subtraindo um membro da igualdade do


outro, temos

(an − cn )bn + (an−1 − cn−1 )bn−1 + . . . + (a1 − c1 )b + (a0 − c0 ) = 0;

logo, deve haver um menor índice i tal que ai − ci ̸= 0. Assim,

bi ((an − cn )bn−i + . . . + (ai+1 − ci+1 )b + (ai − ci )) = 0,

o que nos dá

b((an − cn )bn−i−1 + . . . + (ai+1 − ci+1 )) = (ci − ai ).

Consequentemente, vemos que b | ai − ci . Como 0 ⩽ ai < b e


0 ⩽ ci < b, temos −b < ai − ci < b, o que implica em ai − ci = 0,
contradizendo o fato de ai − ci ̸= 0. Portanto, a representação é mesmo
única. ■
O número b na escrita acima é chamado de base. Comumente, em

112
Capítulo 8. Critérios de Divisibilidade

nosso dia a dia, usamos a base b = 10 para representar números. Em


vez de escrevemos um número a na forma

an bn + an−1 bn−1 + . . . + a2 b2 + a1 b + a0 ,

usamos a concatenação an an−1 . . . a2 a1 a0 . Por exemplo, o número


347 é a concatenação de 3 · 102 + 4 · 10 + 7, na representação decimal.
A base 10 que usamos é composta dos algarismos 0, 1, 2, 3, 4, 5, 7, 7, 8 e 9,
e qualquer número menor do que 10 é presentado por um deste algaris-
mos. Por exemplo, o número 6 é representado pelo próprio algarismo
6.
Na computação, a base usada é a 2, formada pelos algarismo 0 e 1.
Neste texto, sempre que a base não for 10 a indicaremos com uma
subscrição no número. Por exemplo, o símbolo 342 = 100010 significa
a representação do número 34 na base 2.

Exemplo 8.1.1 Representemos o número 14 na base 2.

Solução Utilizaremos o algoritmo da divisão sucessivamente, como


segue,

14 = 2(7) + 0
7 = 2(3) + 1
3 = 2(1) + 1
1 = 2(0) + 1.

Assim, 152 = 1 · 23 + 1 · 22 + 1 · 2 + 0, ou 152 = 1110. ■

Exemplo 8.1.2 Representemos o número 2834 na base 8.

113
Capítulo 8. Critérios de Divisibilidade

Solução Temos

2834 = 8(354) + 2
354 = 8(44) + 2
44 = 8(5) + 5
5 = 8(0) + 5.

Assim, 28348 = 5 · 83 + 5 · 82 + 2 · 8 + 2, ou 28348 = 5522. ■


Se a base for maior do que 10 é conveniente usa outros símbolos
além dos algarismos hindu-arábicos. Por exemplo, se tomarmos a base
12, usaremos os símbolos α e β para representar, respectivamente, 10 e
11.

Exemplo 8.1.3 Representemos o número 5031 na base 12.

Solução Temos

5031 = 12(419) + 3
419 = 12(34) + 11
34 = 12(2) + 10
2 = 12(0) + 2.

Assim, 496712 = 2 · 123 + α · 122 + β · 12 + 3, ou 496712 = 2αβ3. ■

Exercícios

Exercício 8.1.1 Represente o número 123 nas bases 5, 8 e 9.

Exercício 8.1.2 O número 100011101 está na base 2, represente-o na


base 10.

114
Capítulo 8. Critérios de Divisibilidade

Exercício 8.1.3 Represente o número 1425 na base 10.

Exercício 8.1.4 Represente o número 6011257 na base 10.

Exercício 8.1.5 Determine a soma 1325 + 4135 .

Exercício 8.1.6 Represente 2n − 1 na base 2.

8.2 Critérios de Divisibilidade

Para provarmos alguns critérios de divisibilidade usaremos o seguinte


resultado.

Proposição 8.2.1 Seja P(x) = an xn + an−1 xn−1 + . . . + a2 x2 + a1 x + a0


um polinômio com coeficientes inteiros. Se a ≡ b (mod m), então
P(a) ≡ P(b) (mod m).

Prova De fato, sabemos que a ≡ b (mod m) implica em ai ≡ bi


(mod m), para i = 0, 1, 2, . . . , n; assim, ai ai ≡ ai bi (mod m). Agora,
Pn i
Pn i
somando estas n + 1 congruências, temos i=0 ai a ≡ i=0 ai b
(mod m), isto é, P(a) ≡ P(b) (mod m). ■
Como consequência do resultado acima vemos que, se a ≡ b
(mod m) e a for uma solução de P(x) ≡ 0 (mod m), então b também
será uma solução de P(x) ≡ 0 (mod m). De fato, uma vez que a ≡ b
(mod m), temos P(a) ≡ P(b) (mod m), Logo, se tivermos P(a) ≡ 0
(mod m) também temos P(b) ≡ 0 (mod m).

Teorema 8.2.1 (Divisibilidade por 3) Seja a = an an−1 . . . a2 a1 a0 um


número inteiro. Então, a é divisível por 3 se, e somente se, 3 divide
an + an−1 + . . . + a2 + a1 + a0 .

115
Capítulo 8. Critérios de Divisibilidade

Prova Escrevamos a = an 10n + an−1 10n−1 + . . . + a2 102 + a1 10 + a0 .


P
Consideremos o polinômio P(x) = n i
i=0 ai x e notemos que 10 ≡ 1
(mod 3), logo P(10) ≡ P(1) (mod 3). Como P(10) = a e P(1) =
an +an−1 +. . .+a2 +a1 +a0 , temos a ≡ an +an−1 +. . .+a2 +a1 +a0
(mod 3). Segue-se que a ≡ 0 (mod m) se, e somente se, an + an−1 +
. . . + a2 + a1 + a0 ≡ 0 (mod 3). ■

Exemplo 8.2.1 O número 369 é divisível por 3, pois 3 | 3 + 6 + 9 = 18.

Teorema 8.2.2 (Divisibilidade por 4) Seja a = an an−1 . . . a2 a1 a0 um


número inteiro. Então, a é divisível por 4 se, e somente se, 4 divide
10a1 + a0 = a1 a0 .

Prova Escrevamos a = an 10n + an−1 10n−1 + . . . + a2 102 + a1 10 + a0 .


Pn i i
Consideremos o polinômio P(x) = i=0 ai x e notemos que 10 ≡
0 (mod 4), para i ⩾ 2; além disso, P(10) ≡ P(10) (mod 4). Como
P(10) = a e P(10) ≡ 10a1 + a0 (mod 4), segue-se que a ≡ 0 (mod 4)
se, e somente se, 10a1 + a0 = a1 a0 ≡ 0 (mod 4). ■

Exemplo 8.2.2 O número 1536 é divisível por 4, pois 4 | 36.

Teorema 8.2.3 (Divisibilidade por 7) Seja a = an an−1 . . . a2 a1 a0 um


número inteiro. Então, a é divisível por 7 se, e somente se, 7 divide
an an−1 . . . a2 a1 − 2a0 .

Prova Escrevamos a = an 10n +an−1 10n−1 +. . .+a2 102 +a1 10+a0 =


10(an 10n−1 + an−1 10n−2 + . . . + a1 − 2a0 ) + 21a0 . Consideremos o
Pn i
polinômio P(x) = i=0 ai x . Como 10 ≡ 10 (mod 7), vemos que
P(10) ≡ P(10) (mod 7). Porém, P(10) = a, assim, P(10) ≡ 0 (mod 7)
se, e somente, se 10(an 10n−1 + an−1 10n−2 + . . . + a1 − 2a0 ) ≡ 0
(mod 7) se, e somente se, an 10n−1 + an−1 10n−2 + . . . + a1 − 2a0 ≡ 0

116
Capítulo 8. Critérios de Divisibilidade

(mod 7). Uma vez que an 10n−1 + an−1 10n−2 + . . . + a1 − 2a0 =


an an−1 . . . a1 − 2a0 , concluímos o resultado. ■

Exemplo 8.2.3 O número 343 é divisível por 7, pois 7 | 34 − 2 · 3 = 28.

Teorema 8.2.4 (Divisibilidade por 11) Seja a = an an−1 . . . a2 a1 a0 um


número inteiro. Então, a é divisível por 11 se, e somente se, 7 divide
(−1)n an + (−1)n−1 an−1 + . . . + a2 − a1 + a0 .

Prova Escrevamos a = an 10n + an−1 10n−1 + . . . + a2 102 + a1 10 + a0 .


P
Consideremos o polinômio P(x) = n i
i=0 ai x e notemos que 10 ≡ −1
(mod 11); logo, P(10) ≡ P(−1) (mod 11). Como P(10) = a e P(−1) =
(−1)n an + (−1)n−1 an−1 + . . . + a2 − a1 + a0 , vemos que P(10) ≡ 0
(mod 11) se, e somente, se P(−1) = (−1)n an + (−1)n−1 an−1 + . . . +
a2 − a1 + a0 ≡ 0 (mod 11). ■

Exemplo 8.2.4 O número 105204 é divisível por 11, pois 11 | 1 − 0 + 5 −


2 + 0 − 4 = 0.

Exercícios

Exercício 8.2.1 Mostre que um inteiro a = an an−1 . . . a2 a1 a0 é divisível


por 2 se, e somente se, a0 = 0, 2, 4, 6 ou 8.

Exercício 8.2.2 Mostre que um inteiro a = an an−1 . . . a2 a1 a0 é divisível


por 5 se, e somente se, a0 = 0 ou 5.

Exercício 8.2.3 Mostre que um inteiro a = an an−1 . . . a2 a1 a0 é divisível


por 6 se, e somente se, a é divisível por 2 e por 3.

Exercício 8.2.4 Mostre que um inteiro a = an an−1 . . . a2 a1 a0 é divisível


por 7 se, e somente se, an an−1 . . . a2 a1 + 5a0 é divisível por 7.

117
Capítulo 8. Critérios de Divisibilidade

Exercício 8.2.5 Mostre que um inteiro a = an an−1 . . . a2 a1 a0 é divisível


por 8 se, e somente se, a2 a1 a0 é divisível por 8.

Exercício 8.2.6 Mostre que um inteiro a = an an−1 . . . a2 a1 a0 é divisível


por 9 se, e somente se, an + an−1 + . . . a2 + a1 + a0 é divisível por 9.

Exercício 8.2.7 Encontre um critério de divisibilidade por 10, 12, 13 e


25.

Exercício 8.2.8 Mostre que um inteiro a = an an−1 . . . a2 a1 a0 é divisível


por 6 se, e somente se, 4an + 4an−1 + . . . 4a2 + 4a1 + a0 é divisível
por 6.

118
9
Fermat e Wilson

Neste capítulo estudaremos o Pequeno Teorema de Fermat e o


Teorema de Wilson.

9.1 Teorema de Fermat

Consideremos o primo p = 7 e o inteiro a = 3. Notemos que 1·3 ≡ 3


(mod 7), 2·3 ≡ 6 (mod 7), 3·3 ≡ 2 (mod 7), 4·3 ≡ 5 (mod 7), 5·3 ≡ 1
(mod 7) e 6 · 3 ≡ 4 (mod 7). Multiplicando essas congruências vemos
que 36 · 6! ≡ 6! (mod 7), o que implica em 36 ≡ 1 (mod 7). Em outras
palavras, ap−1 ≡ 1 (mod p). Propomos o seguinte teorema.

Teorema 9.1.1 (Pequeno Teorema de Fermat) Seja a um número inteiro.


Se p é um primo tal que p ∤ a, então ap−1 ≡ 1 (mod p).

Prova Consideremos os seguintes p − 1 múltiplos de a:

a, 2a, . . . , (p − 1)a.

Note que nenhum desses inteiros é divisível por p. De fato, um vez


Capítulo 9. Fermat e Wilson

que 1, 2, . . . , p − 1 é um sistema reduzido de resíduo módulo p, conse-


quentemente, a, 2a, . . . , (p − 1)a também é. Em outras palavras, os
inteiros a, 2a, . . . , (p − 1)a são dois a dois incongruentes módulo p.
Em consequência disso, um dos inteiros a, 2a, . . . , (p − 1)a deve ser
congruente, módulo p, a algum dos inteiros 1, 2, . . . , p − 1, em alguma
ordem. Logo,

a · 2a · . . . · (p − 1)a ≡ 1 · 2 · . . . · (p − 1) (mod p),

ou ap−1 (p − 1)! ≡ (p − 1)! (mod p). Como ((p − 1)!, p) = 1, pela


proposição 7.2.2, podemos cancelar o fator (p − 1)!, o que nos dá
ap−1 ≡ 1 (mod p). ■

Corolário 9.1.1 (Teorema de Fermat) Seja a um número inteiro. Se p é


primo então ap ≡ a (mod p).

Prova Digamos que p ∤ a. Então pelo Pequeno Teorema de Fermat,


temos ap−1 ≡ 1 (mod p); além disso a ≡ a (mod p). Multiplicando
as duas congruências obtemos ap ≡ a (mod p).
Se agora p | a, então p | ap ; logo, a ≡ 0 (mod p) e ap ≡ 0 (mod p).
Deste modo, ap ≡ a (mod p). ■

Exemplo 9.1.1 Mostremos que o resto da divisão de 3201 por 11 é 3.

Solução Primeiramente, notemos que 310 ≡ 1 (mod 11). Além disso,


201 = 10(20) + 1, logo, 3201 = 310(20)+1 = (310 )20 · 3. Assim, vemos
que 3201 ≡ 3 (mod 11). ■

120
Capítulo 9. Fermat e Wilson

Exercícios

Exercício 9.1.1 Verifique que 1 é o resto o menor encontrado na divisão


de 11104 + 2 por 17.

Exercício 9.1.2 Mostre que 310 − 1 é divisível por 112 .

Exercício 9.1.3 Determine o resto da divisão de 5100 por 7.

Exercício 9.1.4 Determine o resto da divisão de 62000 por 11.

Exercício 9.1.5 Se (a, 35) = 1, mostre que a12 ≡ 1 (mod 35).

Exercício 9.1.6 Mostre que a21 ≡ a (mod 15), para todo a.

Exercício 9.1.7 Mostre que a7 ≡ a (mod 42), para todo a.

Exercício 9.1.8 Mostre que a9 ≡ a (mod 30), para todo a.

Exercício 9.1.9 Mostre que, se p é primo e a é um inteiro tal que p ∤ a,


então a e ap−2 são inversos módulo p.

Exercício 9.1.10 Mostre que, se p é primo e a, b são inteiros tais que


p ∤ a, então as soluções da congruência linear ax ≡ b (mod p) são os
inteiros x tais que x ≡ ap−2 b (mod p).

Exercício 9.1.11 Determine as soluções de

a) 7x ≡ 12 (mod 17);

b) 4x ≡ 11 (mod 19).

Exercício 9.1.12 Mostre que, se p, q são primos distintos tais que ap ≡ a


(mod p) e aq ≡ a (mod p), então apq ≡ a (mod p).

Exercício 9.1.13 Sejam a, b inteiros não divisíveis pelo primo p. Mostre


que, se ap ≡ bp (mod p), então ap ≡ bp (mod p2 )

121
Capítulo 9. Fermat e Wilson

9.2 Teorema de Wilson

Consideremos p = 7. Notemos que 2 · 4 ≡ 1 (mod 7) e 3 · 5 ≡ 1


(mod 7). Como 6 ≡ 6 (mod 7), vemos que 6! ≡ 6 ≡ −1 (mod 7). Em
outras palavras (p − 1)! ≡ −1 (mod p). Propomos o seguinte teorema.

Teorema 9.2.1 (Teorema de Wilson) Seja p um número primo. Então


(p − 1)! ≡ −1 (mod p).

Prova Os casos em que p = 2 ou 3 são fáceis de perceber, suponhamos


assim, p > 3. Seja a algum dos inteiros

1, 2, . . . , p − 1

e consideremos a congruência linear ax ≡ 1 (mod p). Como (a, p) =


1 | 1, essa congruência tem única solução módulo p. Para cada um
dos inteiros 1, 2, . . . , p − 1 há um único inverso a ′ , tal que aa ′ ≡ 1
(mod p). Dentre estes, o únicos que são seus próprios inversos são 1
e p − 1, pois a2 ≡ 1 (mod p), equivale a (a − 1)(a + 1) ≡ 0 (mod p).
Assim, a − 1 ≡ 0 (mod p) ou a + 1 ≡ 0 (mod p). No primeiro caso,
temos a = 1, e no segundo, a = p − 1. Agora, agrupando aos pares os
inteiros de 2 a p − 2 tais que o produto de cada par seja congruente a 1
módulo p, vemos que

(p − 2) · (p − 3) · . . . · 3 · 2 ≡ 1 (mod p),

ou (p − 2)! ≡ 1 (mod p). Por fim, multiplica essa última congruência


por 1 e p − 1, obtemos (p − 1)(p − 2)! ≡ p − 1 ≡ −1 (mod p), o que
conclui nosso resultado. ■

Exemplo 9.2.1 Mostremos, usando o mesmo argumento da demonstração

122
Capítulo 9. Fermat e Wilson

do teorema acima que 10! ≡ −1 (mod 11).

Solução Primeiramente, notemos que

2·6≡1 (mod 11)


3·4≡1 (mod 11)
5·9≡1 (mod 11)
7·8≡1 (mod 11).

Assim, 10! = (2 · 6)(3 · 4)(5 · 9)(7 · 8) ≡ 10 ≡ −1 (mod 11). ■


O teorema de Wilson é um teste de primalidade, e vale a sua
recíproca.

Teorema 9.2.2 Seja n um inteiro. Se (n − 1)! ≡ −1 (mod n), então n é


primo.

Prova Digamos que (n − 1)! ≡ −1 (mod n) e n não é primo, então n


tem algum divisor d tal que 1 < d < n. Como d ⩽ n − 1, vemos que d
é um fator de (n − 1)!, logo d | (n − 1)!. Uma vez que (n − 1)! ≡ −1
(mod n), temos n | (n − 1)! + 1. Assim é evidente que d | (n − 1)! + 1;
logo, d | 1, o que é um absurdo, pois 1 < d. Portanto, n é primo. ■

Exemplo 9.2.2 Mostremos que 6 não é primo.

Solução Como (6 − 1)! = 5! = 120 ≡ 0 (mod 6), vemos pelo Teorema


de Wilson que 6 não é primo. ■

Exercícios

Exercício 9.2.1 Determine o resto da divisão de 15! por 17.

Exercício 9.2.2 Determine o resto da divisão de 21! por 23.

123
Capítulo 9. Fermat e Wilson

Exercício 9.2.3 Verifique que 13 é primo concluindo que 12! ≡ −1


(mod 13).

Exercício 9.2.4 Mostre que 18! ≡ −1 (mod 437).

Exercício 9.2.5 Mostre que n > 1 é primo se, e somente se, (n − 2)! ≡ 1
(mod n).

Exercício 9.2.6 Mostre que, se p é um primo ímpar, então 2(p − 2)! ≡ 1


(mod p).

124
10
Euler

Neste capítulo veremos a generalização do Pequeno Teorema de


Fermat dada por Euler.

10.1 Função de Euler

Definição 10.1.1 (φ de Euler) Seja n um número natural. Chamamos


de fi de Euler a função φ que conta o número de inteiros positivos
menores do que ou iguais a n primos com n.

O símbolo φ é uma letra grega denominada fi.

Exemplo 10.1.1 Se n = 20, temos φ(20) = 8, pois os números menores


do que ou iguais a 20 primos com 20 são 1, 3, 7, 9, 11, 13, 17 e 19.

Decorre imediatamente, da definição acima, o seguinte resultado.

Proposição 10.1.1 Se n = p é um número primo, então φ(p) = p − 1.

Prova De fato, na sequência 1, 2, . . . , n há n números. Caso n = p seja


primo, deste apenas (p, p) ̸= 1; assim, restam p − 1 números primos
Capítulo 10. Euler

com p. Portanto, φ(p) = p − 1. ■


Mostraremos agora que a função φ é multiplicativa. Para isso,
usarmos um argumento combinatório.

Teorema 10.1.1 (A função φ é multiplicativa) Sejam m, n inteiros positivos,


com (m, n) = 1. Então, φ(mn) = φ(m)φ(n).

Prova Para mostrarmos que φ(mn) = φ(m)φ(n), consideremos a


tabela abaixo, com m linhas e n colunas, de todos os números compre-
endidos entre 1 e mn:

1 m + 1 2m + 1 . . . (n − 1)m + 1
2 m + 2 2m + 2 . . . (n − 1)m + 2
3 m + 3 2m + 3 . . . (n − 1)m + 3
.. .. .. .. ..
. . . . .
r m + r 2m + r . . . (n − 1)m + r
.. .. .. .. ..
. . . . .
m 2m 3m ... mn.

Perceba que, os elementos da linha r são da forma qm + r, com


0 ⩽ q ⩽ (n − 1). Consideremos (m, r) = d. Se d > 1, vemos que
nenhum dos elementos desta linha é primo com mn, pois d | qm + r,
já que d | m e d | r. Assim, para encontrarmos os números na tabela
acima que são primos com mn, precisamos olhar todas as r’s linhas em
que (m, r) = 1.
Se (m, r) = 1 os números r, m + r, 2m + r, . . . , (n − 1)m + r são
todos primos com m, pois (qm + r, r) = 1; logo, há φ(m) linhas
contendo elementos primos com m. Mais ainda, como os números
0, 1, 2, . . . , n − 1 formam um sistema completo de resíduo módulo n, os

126
Capítulo 10. Euler

números r, m + r, 2m + r, . . . , (n − 1)m + r também formam um sistema


completo de resíduo módulo n. Logo, em cada uma das φ(m) linhas, há
um total de φ(n) elementos primos com n. Assim, temos um total de
φ(m)φ(n) números primos com m e n. Portanto, φ(mn) = φ(m)φ(n).

Uma vez que a função φ é multiplicativa somos capazes de encon-
trar uma fórmula que nos permita calcular φ(n) sabendo a fatoração
primária de n. Vejamos primeiramente um caso particular.

Proposição 10.1.2 Se n = pα , então φ(pα ) = pα−1 (p − 1).

Prova Uma vez que pα = p · p · . . . · p é um produto de α fatores de p,


vemos, pela proposição 10.1.1, que

φ(pα ) = φ(p · p · . . . · p)
= φ(p) · φ(p) · . . . · φ(p)
= (p − 1) · (p − 1) · . . . · (p − 1)
= pα−1 (p − 1),

como queríamos. ■

Teorema 10.1.2 Se n = pα α2 αr
1 · p2 · . . . · pr , então
1

φ(n) = pα
1
1 −1
(p1 − 1)pα
2
2 −1
(p2 − 1) . . . prαr −1 (pr − 1).

Prova Uma vez que φ é multiplicativa e φ(pαi αi −1


i ) = pi (pi − 1), vemos
que

φ(n) = p1α1 −1 (p1 − 1)pα


2
2 −1
(p2 − 1) . . . pα
r
r −1
(pr − 1),

127
Capítulo 10. Euler

como queríamos. ■

Exemplo 10.1.2 Para 136 = 23 ·17, vemos que φ(23 ·17) = φ(23 )φ(17) =
23−1 (2 − 1) · 171−1 (17 − 1) = 2 · 16 = 38.

Exemplo 10.1.3 Determinemos os inteiros n tais que φ(n) = 8.

Solução Notemos que nenhum primo maior do que 9 é um fator de


n, caso contrário, φ(n) teria um fator pi − 1 > 8. Mais ainda, 7
também não é um fator de n, pois 7 − 1 = 6 ∤ 8. Assim, os únicos
primos que constam na fatoração primária de n devem ser 2, 3 e 5, isto
é, n = 2α · 3β · 5γ .
Uma vez que n = 2α · 3β · 5γ note que nem β e nem γ podem
assumir valor maior do que 1, senão 3 e 5 seriam fatores de φ(n) = 8.
Logo, basta verificarmos os caso em β = 0 ou 1 e γ = 0 ou 1.
Para β = 0 e γ = 0, temos φ(n) = 2α−1 = 8, o que nos dá α = 4 e
n = 16.
Para β = 0 e γ = 1, temos φ(n) = 2α−1 · 4 = 8, o que nos dá α = 2
e n = 20.
Para β = 1 e γ = 0, temos φ(n) = 2α−1 · 2 = 8, o que nos dá α = 3
e n = 24.
Para β = 1 e γ = 1, temos φ(n) = 2α−1 · 2 · 4 = 8, o que nos dá
α = 1 e n = 30. No entanto, neste caso, temos n = 2α · 3 · 5; assim,
precisamos considerar também α = 0, para n = 15.
Portanto, os n procurados são 15, 16, 20, 24 e 30. ■

Exercícios

Exercício 10.1.1 Calcule φ(n) para n = 100, 256, 1001 e 10!.

128
Capítulo 10. Euler

Exercício 10.1.2 Determine todos os inteiros n tais que φ(n) = 6,


φ(n) = 12 e φ(n) = 24.

Exercício 10.1.3 Mostre que não nenhum inteiro n tal que φ(n) = 14.

Exercício 10.1.4 Mostre que, se n > 2, então φ(n) é par.


[Dica: Considere o caso em que n é um potência de 2 e o caso em que
não é.]

10.2 Teorema de Euler

O Pequeno Teorema de Fermat nos diz que, se p é um número primo


e a é um inteiro tal que (a, p) = 1 a congruência ap−1 ≡ 1 (mod p)
é satisfeita. Daremos uma generalização deste resultado proposta por
Euler. Precisamos antes do seguinte resultado.

Lema 10.2.1 Sejam a, m inteiros, com (a, m) = 1. Se a1 , a2 , . . . , aφ(m)


for um sistema reduzido de resíduo módulo m, então os números
aa1 , aa2 , . . . , aaφ(m) também são um sistema reduzido de resíduo.

Prova É uma consequência do teorema 7.2.1. ■

Teorema 10.2.1 (Teorema de Euler) Sejam a, m inteiros, com (a, m) = 1.


Então aφ(m) ≡ 1 (mod m).

Prova Uma vez que a1 , a2 , . . . , aφ(m) é um sistema reduzido de resíduo


módulo m sabemos que aa1 , aa2 , . . . , aaφ(m) também é. Cada um
dos números aai é congruente módulo m a apenas um dos números ai ,
em alguma ordem. Deste modo,

aa1 a2 . . . aaφ(m) ≡ a1 a2 . . . aφ(m) (mod m);

129
Capítulo 10. Euler

assim,

aφ(m) a1 a2 . . . aφ(m) ≡ a1 a2 . . . aφ(m) (mod m).

Como (a1 a2 . . . aφ(m) , m) = 1, obtemos

aφ(m) ≡ 1 (mod m),

como queríamos. ■

Exemplo 10.2.1 Os conjuntos 1, 5 e 5 · 1, 5 · 5 são sistemas reduzidos de


resíduo módulo 6. Vemos que

(5 · 1)(5 · 5) ≡ 5 · 1 (mod 6),

ou
52 = 5φ(6) ≡ 1 (mod 6).

Note que a e aφ(m)−1 são inversos módulo m, pois a · aφ(m)−1 =


aφ(m) ≡ 1 (mod m). Vemos assim que x é uma solução de ax ≡ b
(mod m) desde que x ≡ aφ(m)−1 b (mod m).

Exemplo 10.2.2 A solução de 3x ≡ 7 (mod 10) é todo x tal que x ≡


3φ(10)−1 · 7 (mod 10). Como φ(10) = 4, temos x ≡ 33 · 7 (mod 10).

Exemplo 10.2.3 Determinemos o resto da divisão de 750 por 9.

Solução Primeiramente, note que φ(9) = φ(32 ) = 6. Uma vez que


(7, 9) = 1, pelo Teorema de Euler, temos 76 ≡ 1 (mod 9). Como
50 = 6(8) + 2 e 72 ≡ 4 (mod 34) vemos que

750 = 76·8+2 = 76·8 · 72 ≡ 72 ≡ 4 (mod 9).

130
Capítulo 10. Euler

Logo, o resto procurado é 4. ■


Algumas vezes somos capazes de encontrar uma potência a. de a
tal que an ≡ 1 (mod m), com (a, m) = 1 e n < φ(m). Por exemplo,
se considerarmos a = 2 e m = 7, vemos que n = 3 < φ(7) = 7 e 23 ≡ 1
(mod 7). Propomos a seguinte definição.

Definição 10.2.1 (Ordem de um número inteiro) Sejam a, m inteiros, tais


que (a, m) = 1. Chamamos de ordem de a módulo m o menor inteiro
positivo n tal que an ≡ 1 (mod m).

Note que a definição de ordem de um inteiro implica que, se a e


b são congruentes módulo m, então eles têm a mesma ordem, pois
a ≡ b implica em an ≡ bn (mod m). Além disso, só faz sentido falar
de ordem de um inteiro a módulo m se (a, m) = 1, pois a congruência
ax ≡ 1 (mod m) não teria solução caso (a, m) > 1.

Teorema 10.2.2 Seja n a ordem de a segundo o módulo m. Então ah ≡ 1


(mod m) se, e somente se, n | h.

Prova Suponhamos que ah ≡ 1 (mod m). Sabemos, pelo algoritmo da


divisão, que podemos escrever h = nq + r, para q, r ∈ Z, com 0 ⩽ r < n.
Assim,

ah = anq+r = (an )q ar ≡ (1)q ar ≡ 1 (mod m),

isto é, ar ≡ 1 (mod m). Como 0 ⩽ r < n, devemos ter r = 0, pois


n sendo a ordem de a módulo m é o menor inteiro positivo tal que
an ≡ 1 (mod m). Logo, h = nq, o implica em n | h.
Suponhamos agora que n | h, isto é, h = nk para algum k ∈ Z.
Como an ≡ 1 (mod m), vemos que ah = (an )k ≡ 1k = 1 (mod m),

131
Capítulo 10. Euler

isto é, ah ≡ 1 (mod m). ■


Em virtude do teorema acima vemos que n | φ(m), sempre que n
for a ordem de um inteiro a segundo o módulo m. Mais ainda, veja que
agora, em vez de gastarmos tempo procurando todas as potências de a
congruentes a 1 módulo m basta que procuremos aquelas de dividem
φ(m).

Exercícios

Exercício 10.2.1 Determine o resto da divisão de 3100 por 10.

Exercício 10.2.2 Determine o resto da divisão de 31000 por 10.

Exercício 10.2.3 Determine o último digito da expansão decimal de


71000 .

Exercício 10.2.4 Determine a ordem de 5 módulo 23.

Exercício 10.2.5 Resolva a congruência 3x ≡ 5 (mod 16), usando o


Teorema de Euler.

Exercício 10.2.6 Seja n a ordem de a segundo o módulo m. Mostre que


ai ≡ aj (mod m) se, e somente, se i ≡ j (mod n).

Exercício 10.2.7 Mostre que, se n é a ordem de a segundo o módulo m,


então a, a2 , . . . , an são incongruentes módulo m.

Exercício 10.2.8 Mostre que, se a1 , a2 , . . . , aφ(m) é um sistema reduzido


de resíduo módulo m, com m ̸= 2, então a1 + a2 + . . . + aφ(m) ≡ 0
(mod m).

Exercício 10.2.9 Mostre que, se (a, b) = 1, então aφ(b) + bφ(a) ≡ 1


(mod ab).

132

Você também pode gostar